797

As I have heard people did not trust Euler when he first discovered the formula (solution of the Basel problem)

math expression: zeta left parenthesis 2 right parenthesis equals sigma summation Underscript k equals 1 Overscript normal infinity Endscripts StartFraction 1 Over k squared EndFraction equals StartFraction pi squared Over 6 EndFraction period

However, Euler was Euler and he gave other proofs.

I believe many of you know some nice proofs of this, can you please share it with us?

Share

49 Answers 49

Reset to default
1
2 Next
364

OK, here's my favorite. I thought of this after reading a proof from the book "Proofs from the book" by Aigner & Ziegler, but later I found more or less the same proof as mine in a paper published a few years earlier by Josef Hofbauer. On Robin's list, the proof most similar to this is number 9 (EDIT: ...which is actually the proof that I read in Aigner & Ziegler).

When math expression: 0 less than x less than pi divided by 2 we have math expression: 0 less than sine x less than x less than tangent x and thus

math expression: StartFraction 1 Over tangent squared x EndFraction less than StartFraction 1 Over x squared EndFraction less than StartFraction 1 Over sine squared x EndFraction period

Note that math expression: 1 divided by tangent squared x equals 1 divided by sine squared x minus 1. Split the interval math expression: left parenthesis 0 comma pi divided by 2 right parenthesis into math expression: 2 Superscript n equal parts, and sum the inequality over the (inner) "gridpoints" math expression: x Subscript k Baseline equals left parenthesis pi divided by 2 right parenthesis dot left parenthesis k divided by 2 Superscript n Baseline right parenthesis:

math expression: sigma summation Underscript k equals 1 Overscript 2 Superscript n Baseline minus 1 Endscripts StartFraction 1 Over sine squared x Subscript k Baseline EndFraction minus sigma summation Underscript k equals 1 Overscript 2 Superscript n Baseline minus 1 Endscripts 1 less than sigma summation Underscript k equals 1 Overscript 2 Superscript n Baseline minus 1 Endscripts StartFraction 1 Over x Subscript k Superscript 2 Baseline EndFraction less than sigma summation Underscript k equals 1 Overscript 2 Superscript n Baseline minus 1 Endscripts StartFraction 1 Over sine squared x Subscript k Baseline EndFraction period

Denoting the sum on the right-hand side by math expression: upper S Subscript n, we can write this as

math expression: upper S Subscript n Baseline minus left parenthesis 2 Superscript n Baseline minus 1 right parenthesis less than sigma summation Underscript k equals 1 Overscript 2 Superscript n Baseline minus 1 Endscripts left parenthesis StartFraction 2 dot 2 Superscript n Baseline Over pi EndFraction right parenthesis squared StartFraction 1 Over k squared EndFraction less than upper S Subscript n Baseline period

Although math expression: upper S Subscript n looks like a complicated sum, it can actually be computed fairly easily. To begin with,

math expression: StartFraction 1 Over sine squared x EndFraction plus StartStartFraction 1 OverOver sine squared left parenthesis StartFraction pi Over 2 EndFraction minus x right parenthesis EndEndFraction equals StartFraction cosine squared x plus sine squared x Over cosine squared x dot sine squared x EndFraction equals StartFraction 4 Over sine squared 2 x EndFraction period

Therefore, if we pair up the terms in the sum math expression: upper S Subscript n except the midpoint math expression: pi divided by 4 (take the point math expression: x Subscript k in the left half of the interval math expression: left parenthesis 0 comma pi divided by 2 right parenthesis together with the point math expression: pi divided by 2 minus x Subscript k in the right half) we get 4 times a sum of the same form, but taking twice as big steps so that we only sum over every other gridpoint; that is, over those gridpoints that correspond to splitting the interval into math expression: 2 Superscript n minus 1 parts. And the midpoint math expression: pi divided by 4 contributes with math expression: 1 divided by sine squared left parenthesis pi divided by 4 right parenthesis equals 2 to the sum. In short,

math expression: upper S Subscript n Baseline equals 4 upper S Subscript n minus 1 Baseline plus 2 period

Since math expression: upper S 1 equals 2, the solution of this recurrence is

math expression: upper S Subscript n Baseline equals StartFraction 2 left parenthesis 4 Superscript n Baseline minus 1 right parenthesis Over 3 EndFraction period

(For example like this: the particular (constant) solution math expression: left parenthesis upper S Subscript p Baseline right parenthesis Subscript n Baseline equals negative 2 divided by 3 plus the general solution to the homogeneous equation math expression: left parenthesis upper S Subscript h Baseline right parenthesis Subscript n Baseline equals upper A dot 4 Superscript n, with the constant A determined by the initial condition math expression: upper S 1 equals left parenthesis upper S Subscript p Baseline right parenthesis Subscript 1 Baseline plus left parenthesis upper S Subscript h Baseline right parenthesis Subscript 1 Baseline equals 2.)

We now have

math expression: StartFraction 2 left parenthesis 4 Superscript n Baseline minus 1 right parenthesis Over 3 EndFraction minus left parenthesis 2 Superscript n Baseline minus 1 right parenthesis less than or equals StartFraction 4 Superscript n plus 1 Baseline Over pi squared EndFraction sigma summation Underscript k equals 1 Overscript 2 Superscript n Baseline minus 1 Endscripts StartFraction 1 Over k squared EndFraction less than or equals StartFraction 2 left parenthesis 4 Superscript n Baseline minus 1 right parenthesis Over 3 EndFraction period

Multiply by math expression: pi squared divided by 4 Superscript n plus 1 and let math expression: n right arrow normal infinity. This squeezes the partial sums between two sequences both tending to math expression: pi squared divided by 6. Voilà!

Share
7
  • 27
    I might add that, as an alternative, one can evaluate the equivalent sum math expression: sigma summation Underscript m equals 0 Overscript normal infinity Endscripts left parenthesis 2 m plus 1 right parenthesis Superscript negative 2 Baseline equals pi squared divided by 8 by summing only over the odd-numbered gridpoints. Then the midpoint math expression: pi divided by 4 never enters the computation, and one gets an even simpler recurrence, of the form math expression: upper T Subscript n Baseline equals 4 upper T Subscript n minus 1.
    – Hans Lundmark
    Oct 30, 2010 at 21:20
  • 8
    @Downvoter: Well, yes, at least from a modern perspective, since we define series using limits. I don't know if Euler thought about it that way. What's your point?
    – Hans Lundmark
    Nov 12, 2011 at 10:13
  • 35
    @Downvoter: it's hard to know whether you're really serious, but if so...Euler probably did more calculus-y things than any other mathematician in history (including Newton and Leibniz).
    – Pete L. Clark
    Mar 4, 2012 at 19:36
  • 29
    @Downvoter Are you confusing Euler with Euclid?
    – Akiva Weinberger
    Sep 30, 2014 at 3:26
  • 19
    @AkivaWeinberger: Just saw this (sorry it's 3 years late), but I must have been, because I'm not sure what else I could've been thinking either...
    – Downvoter
    Feb 19, 2017 at 22:20
246

We can use the function math expression: f left parenthesis x right parenthesis equals x squared with math expression: negative pi less than or equals x less than or equals pi and find its expansion into a trigonometric Fourier series

math expression: StartFraction a 0 Over 2 EndFraction plus sigma summation Underscript n equals 1 Overscript normal infinity Endscripts left parenthesis a Subscript n Baseline cosine n x plus b Subscript n Baseline sine n x right parenthesis comma

which is periodic and converges to math expression: f left parenthesis x right parenthesis in math expression: left bracket negative pi comma pi right bracket.

Observing that math expression: f left parenthesis x right parenthesis is even, it is enough to determine the coefficients

math expression: a Subscript n Baseline equals StartFraction 1 Over pi EndFraction integral Subscript negative pi Superscript pi Baseline f left parenthesis x right parenthesis cosine n x d x n equals 0 comma 1 comma 2 comma 3 comma period period period comma

because

math expression: b Subscript n Baseline equals StartFraction 1 Over pi EndFraction integral Subscript negative pi Superscript pi Baseline f left parenthesis x right parenthesis sine n x d x equals 0 n equals 1 comma 2 comma 3 comma period period period period

For math expression: n equals 0 we have

math expression: a 0 equals StartFraction 1 Over pi EndFraction integral Subscript negative pi Superscript pi Baseline x squared d x equals StartFraction 2 Over pi EndFraction integral Subscript 0 Superscript pi Baseline x squared d x equals StartFraction 2 pi squared Over 3 EndFraction period

And for math expression: n equals 1 comma 2 comma 3 comma period period period we get

math expression: a Subscript n Baseline equals StartFraction 1 Over pi EndFraction integral Subscript negative pi Superscript pi Baseline x squared cosine n x d x

math expression: equals StartFraction 2 Over pi EndFraction integral Subscript 0 Superscript pi Baseline x squared cosine n x d x equals StartFraction 2 Over pi EndFraction times StartFraction 2 pi Over n squared EndFraction left parenthesis negative 1 right parenthesis Superscript n Baseline equals left parenthesis negative 1 right parenthesis Superscript n Baseline StartFraction 4 Over n squared EndFraction comma

because

math expression: integral x squared cosine n x d x equals StartFraction 2 x Over n squared EndFraction cosine n x plus left parenthesis StartFraction x squared Over n EndFraction minus StartFraction 2 Over n cubed EndFraction right parenthesis sine n x period

Thus

math expression: f left parenthesis x right parenthesis equals StartFraction pi squared Over 3 EndFraction plus sigma summation Underscript n equals 1 Overscript normal infinity Endscripts left parenthesis left parenthesis negative 1 right parenthesis Superscript n Baseline StartFraction 4 Over n squared EndFraction cosine n x right parenthesis period

Since math expression: f left parenthesis pi right parenthesis equals pi squared, we obtain

math expression: pi squared equals StartFraction pi squared Over 3 EndFraction plus sigma summation Underscript n equals 1 Overscript normal infinity Endscripts left parenthesis left parenthesis negative 1 right parenthesis Superscript n Baseline StartFraction 4 Over n squared EndFraction cosine left parenthesis n pi right parenthesis right parenthesis

math expression: pi squared equals StartFraction pi squared Over 3 EndFraction plus 4 sigma summation Underscript n equals 1 Overscript normal infinity Endscripts left parenthesis left parenthesis negative 1 right parenthesis Superscript n Baseline left parenthesis negative 1 right parenthesis Superscript n Baseline StartFraction 1 Over n squared EndFraction right parenthesis

math expression: pi squared equals StartFraction pi squared Over 3 EndFraction plus 4 sigma summation Underscript n equals 1 Overscript normal infinity Endscripts StartFraction 1 Over n squared EndFraction period

Therefore

math expression: sigma summation Underscript n equals 1 Overscript normal infinity Endscripts StartFraction 1 Over n squared EndFraction equals StartFraction pi squared Over 4 EndFraction minus StartFraction pi squared Over 12 EndFraction equals StartFraction pi squared Over 6 EndFraction


Second method (available on-line a few years ago) by Eric Rowland. From

math expression: log left parenthesis 1 minus t right parenthesis equals minus sigma summation Underscript n equals 1 Overscript normal infinity Endscripts StartFraction t Superscript n Baseline Over n EndFraction

and making the substitution math expression: t equals e Superscript i x one gets the series expansion

math expression: w equals Log left parenthesis 1 minus e Superscript i x Baseline right parenthesis equals minus sigma summation Underscript n equals 1 Overscript normal infinity Endscripts StartFraction e Superscript i n x Baseline Over n EndFraction equals minus sigma summation Underscript n equals 1 Overscript normal infinity Endscripts StartFraction 1 Over n EndFraction cosine n x minus i sigma summation Underscript n equals 1 Overscript normal infinity Endscripts StartFraction 1 Over n EndFraction sine n x comma

whose radius of convergence is 1. Now if we take the imaginary part of both sides, the RHS becomes

math expression: normal fraktur upper I w equals minus sigma summation Underscript n equals 1 Overscript normal infinity Endscripts StartFraction 1 Over n EndFraction sine n x comma

and the LHS

math expression: normal fraktur upper I w equals arg left parenthesis 1 minus cosine x minus i sine x right parenthesis equals arc tangent StartFraction minus sine x Over 1 minus cosine x EndFraction period

Since

math expression: arc tangent StartFraction minus sine x Over 1 minus cosine x EndFraction equals minus arc tangent StartStartFraction 2 sine StartFraction x Over 2 EndFraction dot cosine StartFraction x Over 2 EndFraction OverOver 2 sine squared StartFraction x Over 2 EndFraction EndEndFraction

math expression: equals minus arc tangent cotangent StartFraction x Over 2 EndFraction equals minus arc tangent tangent left parenthesis StartFraction pi Over 2 EndFraction minus StartFraction x Over 2 EndFraction right parenthesis equals StartFraction x Over 2 EndFraction minus StartFraction pi Over 2 EndFraction comma

the following expansion holds

math expression: StartFraction pi Over 2 EndFraction minus StartFraction x Over 2 EndFraction equals sigma summation Underscript n equals 1 Overscript normal infinity Endscripts StartFraction 1 Over n EndFraction sine n x period left parenthesis asterisk right parenthesis

Integrating the identity math expression: left parenthesis asterisk right parenthesis, we obtain

math expression: StartFraction pi Over 2 EndFraction x minus StartFraction x squared Over 4 EndFraction plus upper C equals minus sigma summation Underscript n equals 1 Overscript normal infinity Endscripts StartFraction 1 Over n squared EndFraction cosine n x period left parenthesis asterisk asterisk right parenthesis

Setting math expression: x equals 0, we get the relation between C and math expression: zeta left parenthesis 2 right parenthesis

math expression: upper C equals minus sigma summation Underscript n equals 1 Overscript normal infinity Endscripts StartFraction 1 Over n squared EndFraction equals minus zeta left parenthesis 2 right parenthesis period

And for math expression: x equals pi, since

math expression: zeta left parenthesis 2 right parenthesis equals 2 sigma summation Underscript n equals 1 Overscript normal infinity Endscripts StartFraction left parenthesis negative 1 right parenthesis Superscript n minus 1 Baseline Over n squared EndFraction comma

we deduce

math expression: StartFraction pi squared Over 4 EndFraction plus upper C equals minus sigma summation Underscript n equals 1 Overscript normal infinity Endscripts StartFraction 1 Over n squared EndFraction cosine n pi equals sigma summation Underscript n equals 1 Overscript normal infinity Endscripts StartFraction left parenthesis negative 1 right parenthesis Superscript n minus 1 Baseline Over n squared EndFraction equals one half zeta left parenthesis 2 right parenthesis equals minus one half upper C period

Solving for C

math expression: upper C equals minus StartFraction pi squared Over 6 EndFraction comma

we thus prove

math expression: zeta left parenthesis 2 right parenthesis equals StartFraction pi squared Over 6 EndFraction period

Note: this 2nd method can generate all the zeta values math expression: zeta left parenthesis 2 n right parenthesis by integrating repeatedly math expression: left parenthesis asterisk asterisk right parenthesis. This is the reason why I appreciate it. Unfortunately it does not work for math expression: zeta left parenthesis 2 n plus 1 right parenthesis.

Note also the

math expression: upper C equals minus StartFraction pi squared Over 6 EndFraction

can be obtained by integrating math expression: left parenthesis asterisk asterisk right parenthesis and substitute

math expression: x equals 0 comma x equals pi

respectively.

Share
9
  • 3
    Would using fractional calculus to integrate math expression: 0.5 times allow you to obtain math expression: zeta left parenthesis 2 n plus 1 right parenthesis?
    – Alice Ryhl
    Feb 17, 2015 at 15:45
  • 4
    Definitely the best answer! Awesome job. I never really understood a proof of this until I read your post.
    – Neil
    Apr 6, 2015 at 5:51
  • 2
    @KristofferRyhl Sorry to revive a year old comment, but... I tried to integrate math expression: left parenthesis asterisk asterisk right parenthesis .5 times and (if I did it correctly) got a denominator of math expression: n Superscript 2.5 so that doesn't work. Integrating math expression: left parenthesis asterisk asterisk right parenthesis once gives us a denominator of math expression: n cubed but that also gives us math expression: s i n left parenthesis n x right parenthesis which equals 0 when math expression: x equals 0 - so that seems to be the why this method doesn't work for math expression: zeta left parenthesis 2 n plus 1 right parenthesis
    – zerosofthezeta
    Apr 27, 2016 at 19:23
  • 1
    @zerosofthezeta I played a bit around with it myself just now, I can see why it only works for math expression: zeta left parenthesis 2 n right parenthesis and not math expression: zeta left parenthesis 2 n plus a right parenthesis for any 0 lt a lt 2: in order to do the trick you need to substitute math expression: x equals something such that math expression: f equals d Superscript n Baseline sine slash d x Superscript n applied as math expression: f left parenthesis n x right parenthesis equals 1, for any integer n. However it is only when the argument to math expression: zeta is even that you can find such an x.
    – Alice Ryhl
    Apr 27, 2016 at 20:01
  • 1
    It is far from clear how you can integrate the expression (*), what doesnt seems to converge uniformly of converge at all for math expression: StartAbsoluteValue x EndAbsoluteValue less than 1.
    – Masacroso
    Sep 15, 2018 at 15:02
208

Here is an other one which is more or less what Euler did in one of his proofs.

The function math expression: sine x where math expression: x element of double struck upper R is zero exactly at math expression: x equals n pi for each integer n. If we factorized it as an infinite product we get

math expression: sine x equals midline horizontal ellipsis left parenthesis 1 plus StartFraction x Over 3 pi EndFraction right parenthesis left parenthesis 1 plus StartFraction x Over 2 pi EndFraction right parenthesis left parenthesis 1 plus StartFraction x Over pi EndFraction right parenthesis x left parenthesis 1 minus StartFraction x Over pi EndFraction right parenthesis left parenthesis 1 minus StartFraction x Over 2 pi EndFraction right parenthesis left parenthesis 1 minus StartFraction x Over 3 pi EndFraction right parenthesis midline horizontal ellipsis equals

math expression: equals x left parenthesis 1 minus StartFraction x squared Over pi squared EndFraction right parenthesis left parenthesis 1 minus StartFraction x squared Over 2 squared pi squared EndFraction right parenthesis left parenthesis 1 minus StartFraction x squared Over 3 squared pi squared EndFraction right parenthesis midline horizontal ellipsis period

We can also represent math expression: sine x as a Taylor series at math expression: x equals 0:

math expression: sine x equals x minus StartFraction x cubed Over 3 factorial EndFraction plus StartFraction x Superscript 5 Baseline Over 5 factorial EndFraction minus StartFraction x Superscript 7 Baseline Over 7 factorial EndFraction plus midline horizontal ellipsis period

Multiplying the product and identifying the coefficient of math expression: x cubed we see that

math expression: StartFraction x cubed Over 3 factorial EndFraction equals x left parenthesis StartFraction x squared Over pi squared EndFraction plus StartFraction x squared Over 2 squared pi squared EndFraction plus StartFraction x squared Over 3 squared pi squared EndFraction plus midline horizontal ellipsis right parenthesis equals x cubed sigma summation Underscript n equals 1 Overscript normal infinity Endscripts StartFraction 1 Over n squared pi squared EndFraction

or

math expression: sigma summation Underscript n equals 1 Overscript normal infinity Endscripts StartFraction 1 Over n squared EndFraction equals StartFraction pi squared Over 6 EndFraction period

Here are two interesting links:

  • Euler's papers;

  • Euler’s Solution of the Basel Problem – The Longer Story an essay on the subject written by Ed Sandifer.

Share
11
  • 45
    This is a very cool peek into the way math was done in the 18th century. I love the total kamikaze approach of the initial assumption, which, as the Sandifer paper discusses on p. 6, is obviously not strictly justifiable. Sandifer gives math expression: e Superscript x Baseline sine x as an alternative function with the same zeroes.
    – user13618
    Feb 11, 2012 at 15:47
  • 28
    @BenCrowell I think that Euler would argue that math expression: e Superscript x Baseline sine x has an infinite-degree zero at math expression: negative normal infinity, requiring math expression: left parenthesis 1 plus StartFraction x Over normal infinity EndFraction right parenthesis Superscript normal infinity to be appended to the infinite product… which is correct, when interpreted correctly.
    – Akiva Weinberger
    Jan 15, 2016 at 17:11
  • 1
    @Ant Append math expression: limit Underscript upper N right arrow normal infinity Endscripts left parenthesis 1 plus StartFraction x Over upper N EndFraction right parenthesis Superscript upper N Baseline equals e Superscript x to the infinite product
    – Akiva Weinberger
    Jun 20, 2016 at 23:08
  • 3
    @Ant I do not think Euler was known for his rigor. :)
    – Akiva Weinberger
    Jun 21, 2016 at 13:09
  • 1
    @Ant This can, like so many things Euler did, be rigorously expressed in the language of Nonstandard Analysis: pick a positive unlimited number math expression: omega element of Superscript asterisk Baseline double struck upper R, i.e. math expression: omega greater than x for all x element of double struck upper R. Then one can show that math expression: left parenthesis 1 plus StartFraction x Over omega EndFraction right parenthesis Superscript omega is infinitesimally close to math expression: exp left parenthesis x right parenthesis equals sigma summation Underscript k element of double struck upper N Endscripts StartFraction 1 Over k factorial EndFraction x Superscript k for all math expression: x element of double struck upper R, and indeed it does have a zero of infinite order at math expression: x equals negative omega. Which math expression: omega you pick is irrelevant, since the behaviour of the function on the reals is the same up to infinitesimal change.
    – Hyperplane
    Jul 20, 2021 at 11:48
162

Define the following series for math expression: x greater than 0

math expression: StartFraction sine x Over x EndFraction equals 1 minus StartFraction x squared Over 3 factorial EndFraction plus StartFraction x Superscript 4 Baseline Over 5 factorial EndFraction minus StartFraction x Superscript 6 Baseline Over 7 factorial EndFraction plus midline horizontal ellipsis period

Now substitute math expression: x equals StartRoot y EndRoot to arrive at

math expression: StartFraction sine StartRoot y EndRoot Over StartRoot y EndRoot EndFraction equals 1 minus StartFraction y Over 3 factorial EndFraction plus StartFraction y squared Over 5 factorial EndFraction minus StartFraction y cubed Over 7 factorial EndFraction plus midline horizontal ellipsis period

if we find the roots of math expression: StartFraction sine StartRoot y EndRoot Over StartRoot y EndRoot EndFraction equals 0 we find that

math expression: y equals n squared pi squared for math expression: n not equals 0 and math expression: n in the integers

With all of this in mind, recall that for a polynomial

math expression: upper P left parenthesis x right parenthesis equals a Subscript n Baseline x Superscript n Baseline plus a Subscript n minus 1 Baseline x Superscript n minus 1 Baseline plus midline horizontal ellipsis plus a 1 x plus a 0 with roots math expression: r 1 comma r 2 comma midline horizontal ellipsis comma r Subscript n Baseline

math expression: StartFraction 1 Over r 1 EndFraction plus StartFraction 1 Over r 2 EndFraction plus midline horizontal ellipsis plus StartFraction 1 Over r Subscript n Baseline EndFraction equals minus StartFraction a 1 Over a 0 EndFraction

Treating the above series for math expression: StartFraction sine StartRoot y EndRoot Over StartRoot y EndRoot EndFraction as polynomial we see that

math expression: StartFraction 1 Over 1 squared pi squared EndFraction plus StartFraction 1 Over 2 squared pi squared EndFraction plus StartFraction 1 Over 3 squared pi squared EndFraction plus midline horizontal ellipsis equals minus StartStartFraction minus StartFraction 1 Over 3 factorial EndFraction OverOver 1 EndEndFraction

then multiplying both sides by math expression: pi squared gives the desired series.

math expression: StartFraction 1 Over 1 squared EndFraction plus StartFraction 1 Over 2 squared EndFraction plus StartFraction 1 Over 3 squared EndFraction plus midline horizontal ellipsis equals StartFraction pi squared Over 6 EndFraction

Share
5
  • 3
    Does the formula you use actually hold for all entire functions defined by power series? Are there conditions that need to be present for this to work? As an entire function is not determined by its roots (e.g. math expression: f left parenthesis z right parenthesis versus math expression: e Superscript g left parenthesis z right parenthesis Baseline f left parenthesis z right parenthesis), is it clear that making such a change wouldn't affect the answer? Or does this rely on Euler's formula for math expression: sine x as an infinite product? This is certainly an interesting idea, but I fear it could be a misleading coincidence.
    – Aaron
    Aug 14, 2011 at 1:45
  • 10
    This is closely related to the method of Euler already described above by AD.
    – user13618
    Feb 11, 2012 at 16:14
  • @BenCrowell Yes, but slightly different anyway, love this one too.. :)
    – AD - Stop Putin -
    Feb 11, 2012 at 17:29
  • @Alfredo Z. Crazy that we think exactly alike (⊙o⊙) Must upvote!
    – Vim
    Feb 20, 2015 at 3:01
  • However I am confused with this problem (take a look at my question here if you don't mind): How to rectify that the fundamental theorem of algebra also holds for an infinite polynomial?
    – Vim
    Feb 20, 2015 at 3:27
146

This method apparently was used by Tom Apostol in math expression: 1983. I will outline the main ideas of the proof, the details can be found in here or this presentation (page math expression: 27)

Consider

math expression: StartLayout 1st Row  with Label left parenthesis 1 right parenthesis EndLabel 1st Column integral Subscript 0 Superscript 1 Baseline integral Subscript 0 Superscript 1 Baseline StartFraction 1 Over 1 minus x y EndFraction d y d x 2nd Column equals integral Subscript 0 Superscript 1 Baseline integral Subscript 0 Superscript 1 Baseline sigma summation Underscript n greater than or equals 0 Endscripts left parenthesis x y right parenthesis Superscript n Baseline d y d x 2nd Row  with Label left parenthesis 2 right parenthesis EndLabel 1st Column Blank 2nd Column equals sigma summation Underscript n greater than or equals 0 Endscripts integral Subscript 0 Superscript 1 Baseline integral Subscript 0 Superscript 1 Baseline x Superscript n Baseline y Superscript n Baseline d y d x 3rd Row  with Label left parenthesis 3 right parenthesis EndLabel 1st Column Blank 2nd Column equals sigma summation Underscript n greater than or equals 1 Endscripts StartFraction 1 Over n squared EndFraction EndLayout

You can verify that the left hand side is indeed math expression: StartFraction pi squared Over 6 EndFraction by letting math expression: x equals u minus v and math expression: y equals v plus u period

Share
1
  • 12
    This proof is provided in the book, Topics in number theory, volume 1, William Judson Leveque, 1956. Read page 122: ia601900.us.archive.org/34/items/in.ernet.dli.2015.134692/…
    – FDP
    Jul 14, 2017 at 9:39
95

I have two favorite proofs. One is the last proof in Robin Chapman's collection; you really should take a look at it.

The other is a proof that generalizes to the evaluation of math expression: zeta left parenthesis 2 n right parenthesis for all n, although I'll do it "Euler-style" to shorten the presentation. The basic idea is that meromorphic functions have infinite partial fraction decompositions that generalize the partial fraction decompositions of rational functions.

The particular function we're interested in is math expression: upper B left parenthesis x right parenthesis equals StartFraction x Over e Superscript x Baseline minus 1 EndFraction, the exponential generating function of the Bernoulli numbers math expression: upper B Subscript n. B is meromorphic with poles at math expression: x equals 2 pi i n comma n element of double struck upper Z, and at these poles it has residue math expression: 2 pi i n. It follows that we can write, a la Euler,

math expression: StartFraction x Over e Superscript x Baseline minus 1 EndFraction equals sigma summation Underscript n element of double struck upper Z Endscripts StartFraction 2 pi i n Over x minus 2 pi i n EndFraction equals sigma summation Underscript n element of double struck upper Z Endscripts minus left parenthesis StartStartFraction 1 OverOver 1 minus StartFraction x Over 2 pi i n EndFraction EndEndFraction right parenthesis period

Now we can expand each of the terms on the RHS as a geometric series, again a la Euler, to obtain

math expression: StartFraction x Over e Superscript x Baseline minus 1 EndFraction equals minus sigma summation Underscript n element of double struck upper Z Endscripts sigma summation Underscript k greater than or equals 0 Endscripts left parenthesis StartFraction x Over 2 pi i n EndFraction right parenthesis Superscript k Baseline equals sigma summation Underscript k greater than or equals 0 Endscripts left parenthesis negative 1 right parenthesis Superscript n plus 1 Baseline StartFraction 2 zeta left parenthesis 2 n right parenthesis Over left parenthesis 2 pi right parenthesis Superscript 2 n Baseline EndFraction x Superscript 2 n

because, after rearranging terms, the sum over odd powers cancels out and the sum over even powers doesn't. (This is one indication of why there is no known closed form for math expression: zeta left parenthesis 2 n plus 1 right parenthesis.) Equating terms on both sides, it follows that

math expression: upper B Subscript 2 n Baseline equals left parenthesis negative 1 right parenthesis Superscript n plus 1 Baseline StartFraction 2 zeta left parenthesis 2 n right parenthesis Over left parenthesis 2 pi right parenthesis Superscript 2 n Baseline EndFraction

or

math expression: zeta left parenthesis 2 n right parenthesis equals left parenthesis negative 1 right parenthesis Superscript n plus 1 Baseline StartFraction upper B Subscript 2 n Baseline left parenthesis 2 pi right parenthesis Superscript 2 n Baseline Over 2 EndFraction

as desired. To compute math expression: zeta left parenthesis 2 right parenthesis it suffices to compute that math expression: upper B 2 equals one sixth, which then gives the usual answer.

Share
7
  • 5
    This is my favorite proof and the one I was going to post, although Qiaochu's explanation is better than mine would have been. :) Instead, I will just add that there's a nice discussion in Concrete Mathematics (2nd edition, pp 285-286) that relates this argument to proof #7 in Robin's list.
    – Mike Spivey
    Oct 30, 2010 at 19:59
  • In your last equation, shouldn't it be math expression: left parenthesis 2 pi right parenthesis Superscript 2 n? See en.wikipedia.org/wiki/Riemann_zeta_function#Specific_values
    – zerosofthezeta
    Nov 29, 2013 at 8:54
  • @evil: yes, thanks for the correction. Edited.
    – Qiaochu Yuan
    Nov 29, 2013 at 22:30
  • Actually, your partial fraction decomposition of math expression: StartFraction x Over normal e Superscript x Baseline minus 1 EndFraction does not converge :/
    – Célestin
    Aug 15, 2017 at 15:52
  • @Phoenix: yes, that's what makes this proof "Euler-style."
    – Qiaochu Yuan
    Aug 16, 2017 at 2:12
84

Here is one more nice proof, I learned it from Grisha Mikhalkin:

Lemma: Let Z be a complex curve in math expression: double struck upper C squared. Let math expression: upper R left parenthesis upper Z right parenthesis subset of double struck upper R squared be the projection of Z onto its real parts and math expression: upper I left parenthesis upper Z right parenthesis the projection onto its complex parts. If these projections are both one to one, then the area of math expression: upper R left parenthesis upper Z right parenthesis is equal to the area of math expression: upper I left parenthesis upper Z right parenthesis.

Proof: There is an obvious map from math expression: upper R left parenthesis upper Z right parenthesis to math expression: upper I left parenthesis upper Z right parenthesis, given by lifting math expression: left parenthesis x 1 comma x 2 right parenthesis element of upper R left parenthesis upper Z right parenthesis to math expression: left parenthesis x 1 plus i y 1 comma x 2 plus i y 2 right parenthesis element of upper Z, and then projecting to math expression: left parenthesis y 1 comma y 2 right parenthesis element of upper I left parenthesis upper Z right parenthesis. We must prove this map has Jacobian 1. WLOG, translate math expression: left parenthesis x 1 comma y 1 comma x 2 comma y 2 right parenthesis to math expression: left parenthesis 0 comma 0 comma 0 comma 0 right parenthesis and let Z obey math expression: partial differential z 2 divided by partial differential z 1 equals a plus b i near math expression: left parenthesis 0 comma 0 right parenthesis. To first order, we have math expression: x 2 equals a x 1 minus b y 1 and math expression: y 2 equals a y 1 plus b x 1. So math expression: y 1 equals left parenthesis a divided by b right parenthesis x 1 minus left parenthesis 1 divided by b right parenthesis x 2 and math expression: y 2 equals left parenthesis a squared plus b squared right parenthesis divided by b x 1 minus left parenthesis a divided by b right parenthesis x 2. So the derivative of math expression: left parenthesis x 1 comma x 2 right parenthesis right arrow from bar left parenthesis y 1 comma y 2 right parenthesis is math expression: Start 2 By 2 Matrix 1st Row 1st Column a divided by b 2nd Column negative 1 divided by b 2nd Row 1st Column left parenthesis a squared plus b squared right parenthesis divided by b 2nd Column negative a divided by b EndMatrix and the Jacobian is 1. QED

Now, consider the curve math expression: e Superscript minus z 1 Baseline plus e Superscript minus z 2 Baseline equals 1, where math expression: z 1 and math expression: z 2 obey the following inequalities: math expression: x 1 greater than or equals 0, math expression: x 2 greater than or equals 0, math expression: negative pi less than or equals y 1 less than or equals 0 and math expression: 0 less than or equals y 2 less than or equals pi.

Given a point on math expression: e Superscript minus z 1 Baseline plus e Superscript minus z 2 Baseline equals 1, consider the triangle with vertices at 0, math expression: e Superscript minus z 1 and math expression: e Superscript minus z 1 Baseline plus e Superscript minus z 2 Baseline equals 1. The inequalities on the y's states that the triangle should lie above the real axis; the inequalities on the x's state the horizontal base should be the longest side.

Projecting onto the x coordinates, we see that the triangle exists if and only if the triangle inequality math expression: e Superscript minus x 1 Baseline plus e Superscript minus x 2 Baseline greater than or equals 1 is obeyed. So math expression: upper R left parenthesis upper Z right parenthesis is the region under the curve math expression: x 2 equals minus log left parenthesis 1 minus e Superscript minus x 1 Baseline right parenthesis. The area under this curve is

math expression: integral Subscript 0 Superscript normal infinity Baseline minus log left parenthesis 1 minus e Superscript negative x Baseline right parenthesis d x equals integral Subscript 0 Superscript normal infinity Baseline sigma summation StartFraction e Superscript minus k x Baseline Over k EndFraction d x equals sigma summation StartFraction 1 Over k squared EndFraction period

Now, project onto the y coordinates. Set math expression: left parenthesis y 1 comma y 2 right parenthesis equals left parenthesis minus theta 1 comma theta 2 right parenthesis for convenience, so the angles of the triangle are math expression: left parenthesis theta 1 comma theta 2 comma pi minus theta 1 minus theta 2 right parenthesis. The largest angle of a triangle is opposite the largest side, so we want math expression: theta 1, math expression: theta 2 less than or equals pi minus theta 1 minus theta 2, plus the obvious inequalities math expression: theta 1, math expression: theta 2 greater than or equals 0. So math expression: upper I left parenthesis upper Z right parenthesis is the quadrilateral with vertices at math expression: left parenthesis 0 comma 0 right parenthesis, math expression: left parenthesis 0 comma pi divided by 2 right parenthesis, math expression: left parenthesis pi divided by 3 comma pi divided by 3 right parenthesis and math expression: left parenthesis pi divided by 2 comma 0 right parenthesis and, by elementary geometry, this has area math expression: pi squared divided by 6.

Share
5
  • 1
    Very nice indeed! (Although it took me a while to understand that the triangle lives in its own complex plane, not related to the math expression: z 1 and math expression: z 2 planes.) But I think it should be math expression: x 1 greater than or equals 0, math expression: x 2 greater than or equals 0, math expression: e Superscript minus x 1 Baseline plus e Superscript minus x 2 Baseline less than or equals 1, and the quadrilateral should have vertices at math expression: left parenthesis 0 comma 0 right parenthesis, math expression: left parenthesis 0 comma pi divided by 2 right parenthesis, math expression: left parenthesis pi divided by 3 comma pi divided by 3 right parenthesis and math expression: left parenthesis pi divided by 2 comma 0 right parenthesis.
    – Hans Lundmark
    Oct 31, 2010 at 9:35
  • Thanks for the corrections! I still think math expression: e Superscript minus x 1 Baseline plus e Superscript minus x 2 Baseline greater than or equals 1 is right, but I've fixed the others.
    – David E Speyer
    Oct 31, 2010 at 12:12
  • Ah, you're right about that one, of course. Sorry.
    – Hans Lundmark
    Oct 31, 2010 at 14:44
  • 2
    I have another comment too, which I posted as a separate answer because it was too long, and also because I wanted to include an image: math.stackexchange.com/questions/8337/…
    – Hans Lundmark
    Nov 1, 2010 at 12:37
  • @DavidSpeyer , do you think your method (or a similar) can be applied here: math.stackexchange.com/questions/1284161/… ?
    – VividD
    May 24, 2015 at 8:00
77
+100

I'll post the one I know since it is Euler's, and is quite easy and stays in math expression: double struck upper R. (I'm guessing Euler didn't have tools like residues back then).

Let

math expression: s equals sine Superscript negative 1 Baseline x

Then

math expression: integral Subscript 0 Superscript StartFraction pi Over 2 EndFraction Baseline s d s equals StartFraction pi squared Over 8 EndFraction

But then

math expression: integral Subscript 0 Superscript 1 Baseline StartFraction sine Superscript negative 1 Baseline x Over StartRoot 1 minus x squared EndRoot EndFraction d x equals StartFraction pi squared Over 8 EndFraction

Since

math expression: sine Superscript negative 1 Baseline x equals integral StartFraction d x Over StartRoot 1 minus x squared EndRoot EndFraction equals x plus one half StartFraction x cubed Over 3 EndFraction plus StartFraction 1 dot 3 Over 2 dot 4 EndFraction StartFraction x Superscript 5 Baseline Over 5 EndFraction plus StartFraction 1 dot 3 dot 5 Over 2 dot 4 dot 6 EndFraction StartFraction x Superscript 7 Baseline Over 7 EndFraction plus midline horizontal ellipsis

We have

math expression: integral Subscript 0 Superscript 1 Baseline StartSet StartFraction d x Over StartRoot 1 minus x squared EndRoot EndFraction integral StartFraction d x Over StartRoot 1 minus x squared EndRoot EndFraction EndSet equals integral Subscript 0 Superscript 1 Baseline left brace x plus one half StartFraction x cubed Over 3 EndFraction StartFraction d x Over StartRoot 1 minus x squared EndRoot EndFraction plus StartFraction 1 dot 3 Over 2 dot 4 EndFraction StartFraction x Superscript 5 Baseline Over 5 EndFraction StartFraction d x Over StartRoot 1 minus x squared EndRoot EndFraction plus midline horizontal ellipsis right brace

But

math expression: integral Subscript 0 Superscript 1 Baseline StartFraction x Superscript 2 n plus 1 Baseline Over StartRoot 1 minus x squared EndRoot EndFraction d x equals StartFraction 2 n Over 2 n plus 1 EndFraction integral Subscript 0 Superscript 1 Baseline StartFraction x Superscript 2 n minus 1 Baseline Over StartRoot 1 minus x squared EndRoot EndFraction d x

which yields

math expression: integral Subscript 0 Superscript 1 Baseline StartFraction x Superscript 2 n plus 1 Baseline Over StartRoot 1 minus x squared EndRoot EndFraction d x equals StartFraction left parenthesis 2 n right parenthesis factorial factorial Over left parenthesis 2 n plus 1 right parenthesis factorial factorial EndFraction

since all powers are odd.

This ultimately produces:

math expression: StartFraction pi squared Over 8 EndFraction equals 1 plus one half one third left parenthesis two thirds right parenthesis plus StartFraction 1 dot 3 Over 2 dot 4 EndFraction one fifth left parenthesis StartFraction 2 dot 4 Over 3 dot 5 EndFraction right parenthesis plus StartFraction 1 dot 3 dot 5 Over 2 dot 4 dot 6 EndFraction one seventh left parenthesis StartFraction 2 dot 4 dot 6 Over 3 dot 5 dot 7 EndFraction right parenthesis midline horizontal ellipsis

math expression: StartFraction pi squared Over 8 EndFraction equals 1 plus StartFraction 1 Over 3 squared EndFraction plus StartFraction 1 Over 5 squared EndFraction plus StartFraction 1 Over 7 squared EndFraction plus midline horizontal ellipsis

Let

math expression: 1 plus StartFraction 1 Over 2 squared EndFraction plus StartFraction 1 Over 3 squared EndFraction plus StartFraction 1 Over 4 squared EndFraction plus midline horizontal ellipsis equals omega

Then

math expression: StartFraction 1 Over 2 squared EndFraction plus StartFraction 1 Over 4 squared EndFraction plus StartFraction 1 Over 6 squared EndFraction plus StartFraction 1 Over 8 squared EndFraction plus midline horizontal ellipsis equals StartFraction omega Over 4 EndFraction

Which means

math expression: StartFraction omega Over 4 EndFraction plus StartFraction pi squared Over 8 EndFraction equals omega

or

math expression: omega equals StartFraction pi squared Over 6 EndFraction

Share
1
  • 6
    @Downvote This is how Euler did it, if you're interested.
    – Pedro
    Feb 12, 2012 at 1:12
70

The most recent issue of The American Mathematical Monthly (August-September 2011, pp. 641-643) has a new proof by Luigi Pace based on elementary probability. Here's the argument.

Let math expression: upper X 1 and math expression: upper X 2 be independent, identically distributed standard half-Cauchy random variables. Thus their common pdf is math expression: p left parenthesis x right parenthesis equals StartFraction 2 Over pi left parenthesis 1 plus x squared right parenthesis EndFraction for math expression: x greater than 0.

Let math expression: upper Y equals upper X 1 divided by upper X 2. Then the pdf of Y is, for math expression: y greater than 0,

math expression: p Subscript upper Y Baseline left parenthesis y right parenthesis equals integral Subscript 0 Superscript normal infinity Baseline x p Subscript upper X 1 Baseline left parenthesis x y right parenthesis p Subscript upper X 2 Baseline left parenthesis x right parenthesis d x equals StartFraction 4 Over pi squared EndFraction integral Subscript 0 Superscript normal infinity Baseline StartFraction x Over left parenthesis 1 plus x squared y squared right parenthesis left parenthesis 1 plus x squared right parenthesis EndFraction d x

math expression: equals StartFraction 2 Over pi squared left parenthesis y squared minus 1 right parenthesis EndFraction left bracket log left parenthesis StartFraction 1 plus x squared y squared Over 1 plus x squared EndFraction right parenthesis right bracket Subscript x equals 0 Superscript normal infinity Baseline equals StartFraction 2 Over pi squared EndFraction StartFraction log left parenthesis y squared right parenthesis Over y squared minus 1 EndFraction equals StartFraction 4 Over pi squared EndFraction StartFraction log left parenthesis y right parenthesis Over y squared minus 1 EndFraction period

Since math expression: upper X 1 and math expression: upper X 2 are equally likely to be the larger of the two, we have math expression: upper P left parenthesis upper Y less than 1 right parenthesis equals 1 divided by 2. Thus

math expression: one half equals integral Subscript 0 Superscript 1 Baseline StartFraction 4 Over pi squared EndFraction StartFraction log left parenthesis y right parenthesis Over y squared minus 1 EndFraction d y period

This is equivalent to

math expression: StartFraction pi squared Over 8 EndFraction equals integral Subscript 0 Superscript 1 Baseline StartFraction minus log left parenthesis y right parenthesis Over 1 minus y squared EndFraction d y equals minus integral Subscript 0 Superscript 1 Baseline log left parenthesis y right parenthesis left parenthesis 1 plus y squared plus y Superscript 4 Baseline plus midline horizontal ellipsis right parenthesis d y equals sigma summation Underscript k equals 0 Overscript normal infinity Endscripts StartFraction 1 Over left parenthesis 2 k plus 1 right parenthesis squared EndFraction comma

which, as others have pointed out, implies math expression: zeta left parenthesis 2 right parenthesis equals pi squared divided by 6.

Share
0
65

This is not really an answer, but rather a long comment prompted by David Speyer's answer. The proof that David gives seems to be the one in How to compute math expression: sigma summation 1 divided by n squared by solving triangles by Mikael Passare, although that paper uses a slightly different way of seeing that the area of the region math expression: upper U 0 (in Passare's notation) bounded by the positive axes and the curve math expression: e Superscript negative x Baseline plus e Superscript negative y Baseline equals 1,

math expression: integral Subscript 0 Superscript normal infinity Baseline minus ln left parenthesis 1 minus e Superscript negative x Baseline right parenthesis d x comma

is equal to math expression: sigma summation Underscript n greater than or equals 1 Endscripts StartFraction 1 Over n squared EndFraction.

This brings me to what I really wanted to mention, namely another curious way to see why math expression: upper U 0 has that area; I learned this from Johan Wästlund. Consider the region math expression: upper D Subscript upper N illustrated below for math expression: upper N equals 8:

Although it's not immediately obvious, the area of math expression: upper D Subscript upper N is math expression: sigma summation Underscript n equals 1 Overscript upper N Endscripts StartFraction 1 Over n squared EndFraction. Proof: The area of math expression: upper D 1 is 1. To get from math expression: upper D Subscript upper N to math expression: upper D Subscript upper N plus 1 one removes the boxes along the top diagonal, and adds a new leftmost column of rectangles of width math expression: 1 divided by left parenthesis upper N plus 1 right parenthesis and heights math expression: 1 divided by 1 comma 1 divided by 2 comma ellipsis comma 1 divided by upper N, plus a new bottom row which is the "transpose" of the new column, plus a square of side math expression: 1 divided by left parenthesis upper N plus 1 right parenthesis in the bottom left corner. The kth rectangle from the top in the new column and the kth rectangle from the left in the new row (not counting the square) have a combined area which exactly matches the kth box in the removed diagonal:

math expression: StartFraction 1 Over k EndFraction StartFraction 1 Over upper N plus 1 EndFraction plus StartFraction 1 Over upper N plus 1 EndFraction StartFraction 1 Over upper N plus 1 minus k EndFraction equals StartFraction 1 Over k EndFraction StartFraction 1 Over upper N plus 1 minus k EndFraction period

Thus the area added in the process is just that of the square, math expression: 1 divided by left parenthesis upper N plus 1 right parenthesis squared. Q.E.D.

(Apparently this shape somehow comes up in connection with the "random assignment problem", where there's an expected value of something which turns out to be math expression: sigma summation Underscript n equals 1 Overscript upper N Endscripts StartFraction 1 Over n squared EndFraction.)

Now place math expression: upper D Subscript upper N in the first quadrant, with the lower left corner at the origin. Letting math expression: upper N right arrow normal infinity gives nothing but the region math expression: upper U 0: for large N and for math expression: 0 less than alpha less than 1, the upper corner of column number math expression: left ceiling alpha upper N right ceiling in math expression: upper D Subscript upper N lies at

math expression: left parenthesis x comma y right parenthesis equals left parenthesis sigma summation Underscript n equals left ceiling left parenthesis 1 minus alpha right parenthesis upper N right ceiling Overscript upper N Endscripts StartFraction 1 Over n EndFraction comma sigma summation Underscript n equals left ceiling alpha upper N right ceiling Overscript upper N Endscripts StartFraction 1 Over n EndFraction right parenthesis tilde left parenthesis ln StartFraction 1 Over 1 minus alpha EndFraction comma ln StartFraction 1 Over alpha EndFraction right parenthesis comma

hence (in the limit) on the curve math expression: e Superscript negative x Baseline plus e Superscript negative y Baseline equals 1.

Share
7
  • 10
    That's a neat observation.
    – David E Speyer
    Nov 1, 2010 at 14:53
  • 6
    Tracing through the proof that math expression: upper D Subscript upper N Baseline equals sigma summation Underscript d equals 1 Overscript upper N Endscripts 1 divided by d squared, I discovered the following curiosity: If you look at all the rectangle in math expression: upper D Subscript upper N of the form math expression: 1 divided by j times 1 divided by k with math expression: upper G upper C upper D left parenthesis j comma k right parenthesis equals d, their total area is math expression: 1 divided by d squared. In particular, if you look at the rectangles of the form math expression: 1 divided by j times 1 divided by k with math expression: upper G upper C upper D left parenthesis x comma y right parenthesis equals 1, in the limit they are spread everywhere across the region math expression: e Superscript negative x Baseline plus e Superscript negative y Baseline greater than or equals 1, with density equal to the probability that two randomly chosen integers are relatively prime, namely math expression: 6 divided by pi squared.
    – David E Speyer
    Aug 22, 2014 at 0:46
  • @DavidSpeyer: That's also a neat observation! :-)
    – Hans Lundmark
    Aug 22, 2014 at 9:43
  • 3
    @VividD: Feel free to try, I'm not going to stop you! ;-)
    – Hans Lundmark
    May 24, 2015 at 8:18
  • 3
    Kiran Kedlaya pointed out to the following to me: In the interval math expression: left bracket 0 comma 1 right bracket, consider all fractions math expression: p divided by q with math expression: q less than or equals upper N. For example, when math expression: upper N equals 3, look at math expression: left parenthesis 0 divided by 1 comma 1 divided by 3 comma 1 divided by 2 comma 2 divided by 3 comma 1 divided by 1 right parenthesis. Look at the blocks of math expression: upper D 3 of size math expression: 1 divided by q times 1 divided by q prime with math expression: upper G upper C upper D left parenthesis q comma q Superscript prime Baseline right parenthesis equals 1. The pairs math expression: left parenthesis q comma q Superscript prime Baseline right parenthesis which occur are precisely the successive denominators. For example, math expression: 1 times 1 divided by 3, math expression: 1 divided by 3 times 1 divided by 2, math expression: 1 divided by 2 times 1 divided by 3, math expression: 1 divided by 3 times 1 in math expression: upper D 3. We have math expression: 1 divided by left parenthesis q q Superscript prime Baseline right parenthesis equals left parenthesis p divided by q right parenthesis minus left parenthesis p prime divided by q Superscript prime Baseline right parenthesis (this is a well known property of Farey fractions) so the areas add up to 1 because this is the length of the interval.
    – David E Speyer
    Jul 8, 2016 at 19:35
64

Note that

math expression: StartFraction pi squared Over sine squared pi z EndFraction equals sigma summation Underscript n equals negative normal infinity Overscript normal infinity Endscripts StartFraction 1 Over left parenthesis z minus n right parenthesis squared EndFraction

from complex analysis and that both sides are analytic everywhere except math expression: n equals 0 comma plus or minus 1 comma plus or minus 2 comma midline horizontal ellipsis. Then one can obtain

math expression: StartFraction pi squared Over sine squared pi z EndFraction minus StartFraction 1 Over z squared EndFraction equals sigma summation Underscript n equals 1 Overscript normal infinity Endscripts StartFraction 1 Over left parenthesis z minus n right parenthesis squared EndFraction plus sigma summation Underscript n equals 1 Overscript normal infinity Endscripts StartFraction 1 Over left parenthesis z plus n right parenthesis squared EndFraction period

Now the right hand side is analytic at math expression: z equals 0 and hence

math expression: limit Underscript z right arrow 0 Endscripts left parenthesis StartFraction pi squared Over sine squared pi z EndFraction minus StartFraction 1 Over z squared EndFraction right parenthesis equals 2 sigma summation Underscript n equals 1 Overscript normal infinity Endscripts StartFraction 1 Over n squared EndFraction period

Note

math expression: limit Underscript z right arrow 0 Endscripts left parenthesis StartFraction pi squared Over sine squared pi z EndFraction minus StartFraction 1 Over z squared EndFraction right parenthesis equals StartFraction pi squared Over 3 EndFraction period

Thus

math expression: sigma summation Underscript n equals 1 Overscript normal infinity Endscripts StartFraction 1 Over n squared EndFraction equals StartFraction pi squared Over 6 EndFraction period

Share
4
  • 3
    (+1)Nice! The first equality can be derived without complex analysis as well!
    – L. F.
    Mar 8, 2013 at 14:08
  • @L.F. Thank you for telling me the link. It is a short proof without using complex analysis and I like it.
    – xpaul
    Apr 21, 2013 at 17:35
  • (+1) I was considering posting a similar answer based on the proof in this answer of math expression: sigma summation Underscript n equals negative normal infinity Overscript normal infinity Endscripts StartFraction 1 Over z plus n EndFraction equals pi cotangent left parenthesis pi z right parenthesis, then differentiating.
    – robjohn
    Dec 21, 2014 at 17:42
  • 2
    I was about to comment "Curious though how one proves the Weierstrass form of math expression: normal upper Gamma without complex analysis" but it seems the usual proof of Weierstrass factorization for entire functions can be adapted to real analytic functions.
    – Bart Michels
    Oct 28, 2017 at 10:21
64
>

Just as a curiosity, a one-line-real-analytic-proof I found by combining different ideas from this thread and this question:

math expression: StartLayout 1st Row 1st Column zeta left parenthesis 2 right parenthesis 2nd Column equals 3rd Column four thirds sigma summation Underscript n equals 0 Overscript plus normal infinity Endscripts StartFraction 1 Over left parenthesis 2 n plus 1 right parenthesis squared EndFraction equals four thirds integral Subscript 0 Superscript 1 Baseline StartFraction log y Over y squared minus 1 EndFraction d y 2nd Row 1st Column Blank 2nd Column equals 3rd Column two thirds integral Subscript 0 Superscript 1 Baseline StartFraction 1 Over y squared minus 1 EndFraction left bracket log left parenthesis StartFraction 1 plus x squared y squared Over 1 plus x squared EndFraction right parenthesis right bracket Subscript x equals 0 Superscript plus normal infinity d y 3rd Row 1st Column Blank 2nd Column equals 3rd Column four thirds integral Subscript 0 Superscript 1 Baseline integral Subscript 0 Superscript plus normal infinity Baseline StartFraction x Over left parenthesis 1 plus x squared right parenthesis left parenthesis 1 plus x squared y squared right parenthesis EndFraction d x d y 4th Row 1st Column Blank 2nd Column equals 3rd Column four thirds integral Subscript 0 Superscript 1 Baseline integral Subscript 0 Superscript plus normal infinity Baseline StartFraction d x d z Over left parenthesis 1 plus x squared right parenthesis left parenthesis 1 plus z squared right parenthesis EndFraction equals four thirds dot StartFraction pi Over 4 EndFraction dot StartFraction pi Over 2 EndFraction equals StartFraction pi squared Over 6 EndFraction period EndLayout


Update. By collecting pieces, I have another nice proof. By Euler's acceleration method or just an iterated trick like my math expression: left parenthesis 1 right parenthesis here we get:

math expression: StartLayout 1st Row  with Label left parenthesis upper A right parenthesis EndLabel zeta left parenthesis 2 right parenthesis equals sigma summation Underscript n greater than or equals 1 Endscripts StartFraction 1 Over n squared EndFraction equals sigma summation Underscript n greater than or equals 1 Endscripts StartFraction 3 Over n squared StartBinomialOrMatrix 2 n Choose n EndBinomialOrMatrix EndFraction EndLayout

and the last series converges pretty fast. Then we may notice that the last series comes out from a squared arcsine. That just gives another proof of math expression: zeta left parenthesis 2 right parenthesis equals StartFraction pi squared Over 6 EndFraction.


A proof of the identity

math expression: sigma summation Underscript n greater than or equals 0 Endscripts StartFraction 1 Over left parenthesis 2 n plus 1 right parenthesis squared EndFraction equals StartFraction pi Over 2 EndFraction sigma summation Underscript k greater than or equals 0 Endscripts StartFraction left parenthesis negative 1 right parenthesis Superscript k Baseline Over 2 k plus 1 EndFraction equals StartFraction pi Over 2 EndFraction dot StartFraction pi Over 4 EndFraction

is also hidden in tired's answer here. For short, the integral

math expression: upper I equals integral Subscript negative normal infinity Superscript normal infinity Baseline e Superscript y Baseline left parenthesis StartFraction e Superscript y Baseline minus 1 Over y squared EndFraction minus StartFraction 1 Over y EndFraction right parenthesis StartFraction 1 Over e Superscript 2 y Baseline plus 1 EndFraction d y

is clearly real, so the imaginary part of the sum of residues of the integrand function has to be zero.


Still another way (and a very efficient one) is to exploit the reflection formula for the trigamma function:

math expression: psi prime left parenthesis 1 minus z right parenthesis plus psi prime left parenthesis z right parenthesis equals StartFraction pi squared Over sine squared left parenthesis pi z right parenthesis EndFraction

immediately leads to:

math expression: StartFraction pi squared Over 2 EndFraction equals psi prime left parenthesis one half right parenthesis equals sigma summation Underscript n greater than or equals 0 Endscripts StartFraction 1 Over left parenthesis n plus one half right parenthesis squared EndFraction equals 4 sigma summation Underscript n greater than or equals 0 Endscripts StartFraction 1 Over left parenthesis 2 n plus 1 right parenthesis squared EndFraction equals 3 zeta left parenthesis 2 right parenthesis period


2018 update. We may consider that math expression: script upper J equals integral Subscript 0 Superscript plus normal infinity Baseline StartFraction arc tangent x Over 1 plus x squared EndFraction d x equals left bracket one half arc tangent squared x right bracket Subscript 0 Superscript plus normal infinity Baseline equals StartFraction pi squared Over 8 EndFraction.
On the other hand, by Feynman's trick or Fubini's theorem

math expression: script upper J equals integral Subscript 0 Superscript plus normal infinity Baseline integral Subscript 0 Superscript 1 Baseline StartFraction x Over left parenthesis 1 plus x squared right parenthesis left parenthesis 1 plus a squared x squared right parenthesis EndFraction d a d x equals integral Subscript 0 Superscript 1 Baseline StartFraction minus log a Over 1 minus a squared EndFraction d a

and since math expression: integral Subscript 0 Superscript 1 Baseline minus log left parenthesis x right parenthesis x Superscript n Baseline d x equals StartFraction 1 Over left parenthesis n plus 1 right parenthesis squared EndFraction, by expanding math expression: StartFraction 1 Over 1 minus a squared EndFraction as a geometric series we have

math expression: StartFraction pi squared Over 8 EndFraction equals script upper J equals sigma summation Underscript n greater than or equals 0 Endscripts StartFraction 1 Over left parenthesis 2 n plus 1 right parenthesis squared EndFraction period

Share
2
  • 2
    Thanks to @genepeer, I just know that the argument is almost the same of the one presented by Daniele Ritelli and discussed here: euler.genepeer.com/?p=212.
    – Jack D'Aurizio
    Dec 25, 2013 at 9:17
  • 3
    Four lines now, just to make it more readable :)
    – Jack D'Aurizio
    Jul 12, 2014 at 23:24
60

Here is a complex-analytic proof.

For math expression: z element of upper D equals double struck upper C set minus{ math expression: 0 comma 1}, let

math expression: upper R left parenthesis z right parenthesis equals sigma summation StartFraction 1 Over log squared z EndFraction

where the sum is taken over all branches of the logarithm. Each point in D has a neighbourhood on which the branches of math expression: log left parenthesis z right parenthesis are analytic. Since the series converges uniformly away from math expression: z equals 1, math expression: upper R left parenthesis z right parenthesis is analytic on D.

Now a few observations:

(i) Each term of the series tends to 0 as math expression: z right arrow 0. Thanks to the uniform convergence this implies that the singularity at math expression: z equals 0 is removable and we can set math expression: upper R left parenthesis 0 right parenthesis equals 0.

(ii) The only singularity of R is a double pole at math expression: z equals 1 due to the contribution of the principal branch of math expression: log z. Moreover, math expression: limit Underscript z right arrow 1 Endscripts left parenthesis z minus 1 right parenthesis squared upper R left parenthesis z right parenthesis equals 1.

(iii) math expression: upper R left parenthesis 1 divided by z right parenthesis equals upper R left parenthesis z right parenthesis.

By (i) and (iii) R is meromorphic on the extended complex plane, therefore it is rational. By (ii) the denominator of math expression: upper R left parenthesis z right parenthesis is math expression: left parenthesis z minus 1 right parenthesis squared. Since math expression: upper R left parenthesis 0 right parenthesis equals upper R left parenthesis normal infinity right parenthesis equals 0, the numerator has the form math expression: a z. Then (ii) implies math expression: a equals 1, so that

math expression: upper R left parenthesis z right parenthesis equals StartFraction z Over left parenthesis z minus 1 right parenthesis squared EndFraction period

Now, setting math expression: z equals e Superscript 2 pi i w yields

math expression: sigma summation Underscript n equals negative normal infinity Overscript normal infinity Endscripts StartFraction 1 Over left parenthesis w minus n right parenthesis squared EndFraction equals StartFraction pi squared Over sine squared left parenthesis pi w right parenthesis EndFraction

which implies that

math expression: sigma summation Underscript k equals 0 Overscript normal infinity Endscripts StartFraction 1 Over left parenthesis 2 k plus 1 right parenthesis squared EndFraction equals StartFraction pi squared Over 8 EndFraction comma

and the identity math expression: zeta left parenthesis 2 right parenthesis equals pi squared divided by 6 follows.

The proof is due to T. Marshall (American Mathematical Monthly, Vol. 117(4), 2010, P. 352).

Share
1
  • Cool, I never thought about working around the branching problem of math expression: log like this!
    – Bart Michels
    Oct 28, 2017 at 10:18
43

In response to a request here: Compute math expression: contour integral z Superscript minus 2 k Baseline cotangent left parenthesis pi z right parenthesis d z where the integral is taken around a square of side math expression: 2 upper N plus 1. Routine estimates show that the integral goes to 0 as math expression: upper N right arrow normal infinity.

Now, let's compute the integral by residues. At math expression: z equals 0, the residue is math expression: pi Superscript 2 k minus 1 Baseline q, where q is some rational number coming from the power series for math expression: cotangent. For example, if math expression: k equals 1, then we get math expression: negative pi divided by 3.

At math expression: m pi, for math expression: m not equals 0, the residue is math expression: z Superscript minus 2 k Baseline pi Superscript negative 1. So

math expression: pi Superscript negative 1 Baseline limit Underscript upper N right arrow normal infinity Endscripts sigma summation Underscript negative upper N less than or equals m less than or equals upper N m not equals 0 Endscripts m Superscript minus 2 k Baseline plus pi Superscript 2 k minus 1 Baseline q equals 0

or

math expression: sigma summation Underscript m equals 1 Overscript normal infinity Endscripts m Superscript minus 2 k Baseline equals minus pi Superscript 2 k Baseline q divided by 2

as desired. In particular, math expression: sigma summation m Superscript negative 2 Baseline equals minus left parenthesis pi squared divided by 3 right parenthesis divided by 2 equals pi squared divided by 6.

Common variants: We can replace math expression: cotangent with math expression: tangent, with math expression: 1 divided by left parenthesis e Superscript 2 pi i z Baseline minus 1 right parenthesis, or with similar formulas.

This is reminiscent of Qiaochu's proof but, rather than actually establishing the relation math expression: pi Superscript negative 1 Baseline cotangent left parenthesis pi z right parenthesis equals sigma summation left parenthesis z minus n right parenthesis Superscript negative 1, one simply establishes that both sides contribute the same residues to a certain integral.

Share
36

Another variation. We make use of the following identity (proved at the bottom of this note):

math expression: sigma summation Underscript k equals 1 Overscript n Endscripts cotangent squared left parenthesis StartFraction 2 k minus 1 Over 2 n EndFraction StartFraction pi Over 2 EndFraction right parenthesis equals 2 n squared en dash n period left parenthesis 1 right parenthesis

Now math expression: 1 divided by theta greater than cotangent theta greater than 1 divided by theta minus theta divided by 3 greater than 0 for math expression: 0 less than theta less than pi divided by 2 less than StartRoot 3 EndRoot and so

math expression: 1 divided by theta squared en dash 2 divided by 3 less than cotangent squared theta less than 1 divided by theta squared period left parenthesis 2 right parenthesis

With math expression: theta Subscript k Baseline equals left parenthesis 2 k minus 1 right parenthesis pi divided by 4 n comma summing the inequalities math expression: left parenthesis 2 right parenthesis from math expression: k equals 1 to n we obtain

math expression: 2 n squared en dash n less than sigma summation Underscript k equals 1 Overscript n Endscripts left parenthesis StartFraction 2 n Over 2 k minus 1 EndFraction StartFraction 2 Over pi EndFraction right parenthesis squared less than 2 n squared en dash n plus 2 n divided by 3 period

Hence

math expression: StartFraction pi squared Over 16 EndFraction StartFraction 2 n squared minus n Over n squared EndFraction less than sigma summation Underscript k equals 1 Overscript n Endscripts StartFraction 1 Over left parenthesis 2 k minus 1 right parenthesis squared EndFraction less than StartFraction pi squared Over 16 EndFraction StartFraction 2 n squared minus n divided by 3 Over n squared EndFraction period

Taking the limit as math expression: n right arrow normal infinity we obtain

math expression: sigma summation Underscript k equals 1 Overscript normal infinity Endscripts StartFraction 1 Over left parenthesis 2 k minus 1 right parenthesis squared EndFraction equals StartFraction pi squared Over 8 EndFraction comma

from which the result for math expression: sigma summation Underscript k equals 1 Overscript normal infinity Endscripts 1 divided by k squared follows easily.

To prove math expression: left parenthesis 1 right parenthesis we note that

math expression: cosine 2 n theta equals Re left parenthesis cosine theta plus i sine theta right parenthesis Superscript 2 n Baseline equals sigma summation Underscript k equals 0 Overscript n Endscripts left parenthesis negative 1 right parenthesis Superscript k Baseline StartBinomialOrMatrix 2 n Choose 2 k EndBinomialOrMatrix cosine Superscript 2 n minus 2 k Baseline theta sine Superscript 2 k Baseline theta period

Therefore

math expression: StartFraction cosine 2 n theta Over sine Superscript 2 n Baseline theta EndFraction equals sigma summation Underscript k equals 0 Overscript n Endscripts left parenthesis negative 1 right parenthesis Superscript k Baseline StartBinomialOrMatrix 2 n Choose 2 k EndBinomialOrMatrix cotangent Superscript 2 n minus 2 k Baseline theta period

And so setting math expression: x equals cotangent squared theta we note that

math expression: f left parenthesis x right parenthesis equals sigma summation Underscript k equals 0 Overscript n Endscripts left parenthesis negative 1 right parenthesis Superscript k Baseline StartBinomialOrMatrix 2 n Choose 2 k EndBinomialOrMatrix x Superscript n minus k

has roots math expression: x Subscript j Baseline equals cotangent squared left parenthesis 2 j minus 1 right parenthesis pi divided by 4 n comma for math expression: j equals 1 comma 2 comma ellipsis comma n comma from which math expression: left parenthesis 1 right parenthesis follows since math expression: StartBinomialOrMatrix 2 n Choose 2 n minus 2 EndBinomialOrMatrix equals 2 n squared minus n period

Share
34

A short way to get the sum is to use Fourier's expansion of math expression: x squared in math expression: x element of left parenthesis negative pi comma pi right parenthesis. Recall that Fourier's expansion of math expression: f left parenthesis x right parenthesis is

math expression: ModifyingAbove f With tilde left parenthesis x right parenthesis equals one half a 0 plus sigma summation Underscript n equals 1 Overscript normal infinity Endscripts left parenthesis a Subscript n Baseline cosine n x plus b Subscript n Baseline sine n x right parenthesis comma x element of left parenthesis negative pi comma pi right parenthesis

where

math expression: a 0 equals StartFraction 2 Over pi EndFraction integral Subscript negative pi Superscript pi Baseline f left parenthesis x right parenthesis d x comma a Subscript n Baseline equals StartFraction 2 Over pi EndFraction integral Subscript negative pi Superscript pi Baseline f left parenthesis x right parenthesis cosine n x d x comma b Subscript n Baseline equals StartFraction 2 Over pi EndFraction integral Subscript negative pi Superscript pi Baseline f left parenthesis x right parenthesis sine n x d x comma n equals 1 comma 2 comma 3 comma midline horizontal ellipsis

and

math expression: ModifyingAbove f With tilde left parenthesis x right parenthesis equals StartFraction f left parenthesis x minus 0 right parenthesis plus f left parenthesis x plus 0 right parenthesis Over 2 EndFraction period

Easy calculation shows

math expression: x squared equals StartFraction pi squared Over 3 EndFraction plus 4 sigma summation Underscript n equals 1 Overscript normal infinity Endscripts left parenthesis negative 1 right parenthesis Superscript n Baseline StartFraction cosine n x Over n squared EndFraction comma x element of left bracket negative pi comma pi right bracket period

Letting math expression: x equals pi in both sides gives

math expression: sigma summation Underscript n equals 1 Overscript normal infinity Endscripts StartFraction 1 Over n squared EndFraction equals StartFraction pi squared Over 6 EndFraction period

Another way to get the sum is to use Parseval's Identity for Fourier's expansion of x in math expression: left parenthesis negative pi comma pi right parenthesis. Recall that Parseval's Identity is

math expression: integral Subscript negative pi Superscript pi Baseline StartAbsoluteValue f left parenthesis x right parenthesis EndAbsoluteValue squared d x equals one half a 0 squared plus sigma summation Underscript n equals 1 Overscript normal infinity Endscripts left parenthesis a Subscript n Superscript 2 Baseline plus b Subscript n Superscript 2 Baseline right parenthesis period

Note

math expression: x equals 2 sigma summation Underscript n equals 1 Overscript normal infinity Endscripts left parenthesis negative 1 right parenthesis Superscript n plus 1 Baseline StartFraction sine n x Over n EndFraction comma x element of left parenthesis negative pi comma pi right parenthesis period

Using Parseval's Identity gives

math expression: 4 sigma summation Underscript n equals 1 Overscript normal infinity Endscripts StartFraction 1 Over n squared EndFraction equals integral Subscript negative pi Superscript pi Baseline StartAbsoluteValue x EndAbsoluteValue squared d x

or

math expression: sigma summation Underscript n equals 1 Overscript normal infinity Endscripts StartFraction 1 Over n squared EndFraction equals StartFraction pi squared Over 6 EndFraction period

0

Theorem: Let math expression: StartSet a Subscript n Baseline EndSet be a nonincreasing sequence of positive numbers such that math expression: sigma summation a Subscript n Superscript 2 converges. Then both series

math expression: s colon equals sigma summation Underscript n equals 0 Overscript normal infinity Endscripts left parenthesis negative 1 right parenthesis Superscript n Baseline a Subscript n Baseline comma delta Subscript k Baseline colon equals sigma summation Underscript n equals 0 Overscript normal infinity Endscripts a Subscript n Baseline a Subscript n plus k Baseline comma k element of double struck upper N

converge. Morevere math expression: normal upper Delta colon equals sigma summation Underscript k equals 1 Overscript normal infinity Endscripts left parenthesis negative 1 right parenthesis Superscript k minus 1 Baseline delta Subscript k also converges, and we have the formula

math expression: sigma summation Underscript n equals 0 Overscript normal infinity Endscripts a Subscript n Superscript 2 Baseline equals s squared plus 2 normal upper Delta period

Proof: Knopp. Konrad, Theory and Application of Infinite Series, page 323.

If we let math expression: a Subscript n Baseline equals StartFraction 1 Over 2 n plus 1 EndFraction in this theorem, then we have

math expression: s equals sigma summation Underscript n equals 0 Overscript normal infinity Endscripts left parenthesis negative 1 right parenthesis Superscript n Baseline StartFraction 1 Over 2 n plus 1 EndFraction equals StartFraction pi Over 4 EndFraction

math expression: delta Subscript k Baseline equals sigma summation Underscript n equals 0 Overscript normal infinity Endscripts StartFraction 1 Over left parenthesis 2 n plus 1 right parenthesis left parenthesis 2 n plus 2 k plus 1 right parenthesis EndFraction equals StartFraction 1 Over 2 k EndFraction sigma summation Underscript n equals 0 Overscript normal infinity Endscripts left parenthesis StartFraction 1 Over 2 n plus 1 EndFraction minus StartFraction 1 Over 2 n plus 2 k plus 1 EndFraction right parenthesis equals StartFraction 1 Over 2 k EndFraction left parenthesis 1 plus one third plus period period period plus StartFraction 1 Over 2 k minus 1 EndFraction right parenthesis

Hence,

math expression: sigma summation Underscript n equals 0 Overscript normal infinity Endscripts StartFraction 1 Over left parenthesis 2 n plus 1 right parenthesis squared EndFraction equals left parenthesis StartFraction pi Over 4 EndFraction right parenthesis squared plus sigma summation Underscript k equals 1 Overscript normal infinity Endscripts StartFraction left parenthesis negative 1 right parenthesis Superscript k minus 1 Baseline Over k EndFraction left parenthesis 1 plus one third plus period period period plus StartFraction 1 Over 2 k minus 1 EndFraction right parenthesis equals StartFraction pi squared Over 16 EndFraction plus StartFraction pi squared Over 16 EndFraction equals StartFraction pi squared Over 8 EndFraction

and now

math expression: zeta left parenthesis 2 right parenthesis equals four thirds sigma summation Underscript n equals 0 Overscript normal infinity Endscripts StartFraction 1 Over left parenthesis 2 n plus 1 right parenthesis squared EndFraction equals StartFraction pi squared Over 6 EndFraction period

Share
25

At risk of contravening group etiquette w.r.t. old questions, I'm going to take this opportunity to post my own version. I don't see it in a transparent form in any of the other posts or in Robin Chapman's article, so I invite anyone to point out the correspondence if it's there. I like this argument because it's physical and can be followed without mathematical formalism.

We start by assuming the well-known series for math expression: pi divided by 4 in alternating odd fractions. We can recognize it as the sum of the Fourier series of the square wave, evaluated at the origin:

math expression: cosine left parenthesis x right parenthesis minus cosine left parenthesis 3 x right parenthesis divided by 3 plus cosine left parenthesis 5 x right parenthesis divided by 5 period period period

It is easily argued on physical grounds that this adds up to a square wave; and that the height of the wave is pi/4 follows from the alternating sequence already mentioned. Now we are going to interpret this wave as an electric current flowing through a resistor. There are two ways of calculating the power and they must agree. First, we can just take square of the amplitude; in the case of this square wave, this is obviously a constant and it is just math expression: p i squared divided by 16. The other way is to add up the power of the sinusoidal components. These are the squares of the individual amplitudes:

math expression: 1 plus 1 divided by 9 plus 1 divided by 25 period period period period equals left parenthesis question mark right parenthesis pi squared divided by 16 question mark question mark

No, not quite; I've been a little sloppy and neglected to mention that when calculating the power of a sine wave, you use its RMS amplitude and not its peak amplitude. This introduces a factor of two; so in fact the series as written adds up to math expression: pi squared divided by 8 period This isn't quite what we want; remember we've just added up the odd fractions. But the even fractions contribute in a rather picturesque way; it's easy to group them by powers of two into a geometric sum leading to the desired result of math expression: undefined

Share
3
  • 1
    At the risk of being rude, you've used "It is easily argued on physical grounds" in place of a theorem on pointwise convergence of fourier series, and a particular physical manifestation/application of Plancherel's theorem. You gain "intuition" for why the result is plausible (assuming you have the corresponding physics background), but you lose both rigor and clarity. The problem with making a physical argument for any mathematical fact is that even if you know that certain calculations work for physically relevant examples, it's hard to say what condition "physically relevant" imposes.
    – Aaron
    Aug 14, 2011 at 1:14
  • 2
    Thanks for the feedback. I'm understanding that my argument wasn't so sketchy that you weren't able to fill in the details as necessary. I am blown away by the mathematical sophistication of the people who post hear but I still wish I would see more arguments made the way I do.
    – Marty Green
    Aug 14, 2011 at 1:51
  • 2
    Well, you lucked out that I had seen the argument before (though not phrased with such language), and I remembered enough physics to understand what you were doing. I appreciate how you feel: technical arguments can be difficult to digest and sometimes offer no intuition about the result. A heuristic explanation, even if it isn't fully rigorous, is often a wonderful addition. However, for mathematics, the heuristic cannot be everything, as the mathematical battleground is littered with the bodies of proofs which are simple, intuitive, and wrong.
    – Aaron
    Aug 14, 2011 at 2:08
24

I like this one:

Let math expression: f element of upper L i p left parenthesis upper S Superscript 1 Baseline right parenthesis, where math expression: upper L i p left parenthesis upper S Superscript 1 Baseline right parenthesis is the space of Lipschitz functions on math expression: upper S Superscript 1. So its well defined the number for math expression: k element of double struck upper Z (called Fourier series of f)

math expression: ModifyingAbove f With caret left parenthesis k right parenthesis equals StartFraction 1 Over 2 pi EndFraction integral ModifyingAbove f With caret left parenthesis theta right parenthesis e Superscript minus i k theta Baseline d theta period

By the inversion formula, we have

math expression: f left parenthesis theta right parenthesis equals sigma summation Underscript k element of double struck upper Z Endscripts ModifyingAbove f With caret left parenthesis k right parenthesis e Superscript i k theta Baseline period

Now take math expression: f left parenthesis theta right parenthesis equals StartAbsoluteValue theta EndAbsoluteValue, math expression: theta element of left bracket negative pi comma pi right bracket. Note that math expression: f element of upper L i p left parenthesis upper S Superscript 1 Baseline right parenthesis

We have

math expression:

Using the inversion formula, we have on math expression: theta equals 0 that

math expression: 0 equals sigma summation Underscript k element of double struck upper Z Endscripts ModifyingAbove f With caret left parenthesis k right parenthesis period

Then,

\begin{eqnarray} 0 &=& \frac{\pi}{2}-\sum_{k\in\mathbb{Z}\ |k|\ odd}\frac{2}{k^{2}\pi} \nonumber \\ &=& \frac{\pi}{2}-\sum_{k\in\mathbb{N}\ |k|\ odd}\frac{4}{k^{2}\pi} \nonumber \\ \end{eqnarray}

This implies

math expression: sigma summation Underscript k element of double struck upper N StartAbsoluteValue k EndAbsoluteValue o d d Endscripts StartFraction 1 Over k squared EndFraction equals StartFraction pi squared Over 8 EndFraction

If we multiply the last equation by math expression: StartFraction 1 Over 2 Superscript 2 n Baseline EndFraction with math expression: n equals 0 comma 1 comma 2 comma period period period ,we get

math expression: sigma summation Underscript k element of double struck upper N StartAbsoluteValue k EndAbsoluteValue o d d Endscripts StartFraction 1 Over left parenthesis 2 Superscript n Baseline k right parenthesis squared EndFraction equals StartFraction pi squared Over 2 Superscript 2 n Baseline 8 EndFraction

Now

math expression: sigma summation Underscript n equals 0 comma 1 comma period period period Endscripts left parenthesis sigma summation Underscript k element of double struck upper N StartAbsoluteValue k EndAbsoluteValue o d d Endscripts StartFraction 1 Over left parenthesis 2 Superscript n Baseline k right parenthesis squared EndFraction right parenthesis equals sigma summation Underscript n equals 0 comma 1 comma period period period Endscripts StartFraction pi squared Over 2 Superscript 2 n Baseline 8 EndFraction

The sum in the left is equal to: math expression: sigma summation Underscript k element of double struck upper N Endscripts StartFraction 1 Over k squared EndFraction

The sum in the right is equal to : math expression: StartFraction pi squared Over 6 EndFraction

So we conclude:

math expression: sigma summation Underscript k element of double struck upper N Endscripts StartFraction 1 Over k squared EndFraction equals StartFraction pi squared Over 6 EndFraction

Note: This is problem 9, Page 208 from the boof of Michael Eugene Taylor - Partial Differential Equation Volume 1.

Share
23
+25

Here's a proof based upon periods and the fact that math expression: zeta left parenthesis 2 right parenthesis and math expression: StartFraction pi squared Over 6 EndFraction are periods forming an accessible identity.

The definition of periods below and the proof is from the fascinating introductory survey paper about periods by M. Kontsevich and D. Zagier.

Periods are defined as complex numbers whose real and imaginary parts are values of absolutely convergent integrals of rational functions with rational coefficient over domains in math expression: double struck upper R Superscript n given by polynomial inequalities with rational coefficients.

The set of periods is therefore a countable subset of the complex numbers. It contains the algebraic numbers, but also many of famous transcendental constants.

In order to show the equality math expression: zeta left parenthesis 2 right parenthesis equals StartFraction pi squared Over 6 EndFraction we have to show that both are periods and that math expression: zeta left parenthesis 2 right parenthesis and math expression: StartFraction pi squared Over 6 EndFraction form a so-called accessible identity.

First step of the proof: math expression: zeta left parenthesis 2 right parenthesis and math expression: pi are periods

There are a lot of different proper representations of math expression: pi showing that this constant is a period. In the referred paper above following expressions (besides others) of math expression: pi are stated:

math expression: \begin{align*} \pi= \iint \limits_{x^2+y^2\leq 1}dxdy=\int_{-\infty}^{\infty}\frac{dx}{1+x^2} \end{a ...

showing that math expression: pi is a period. The known representation

math expression: \begin{align*} \zeta(2)=\iint_{0<x<y<1} \frac{dxdy}{(1-x)y} \end{align*} ...

shows that math expression: zeta left parenthesis 2 right parenthesis is also a period.

Second step: math expression: zeta left parenthesis 2 right parenthesis and math expression: StartFraction pi squared Over 6 EndFraction form an accessible identity.

An accessible identity between two periods A and B is given, if we can transform the integral representation of period A by application of the three rules: Additivity (integrand and domain), Change of variables and Newton-Leibniz formula to the integral representation of period B.

This implies equality of the periods and the job is done.

In order to show that math expression: zeta left parenthesis 2 right parenthesis and math expression: StartFraction pi squared Over 6 EndFraction are accessible identities we start with the integral I

math expression: upper I equals integral Subscript 0 Superscript 1 Baseline integral Subscript 0 Superscript 1 Baseline StartFraction 1 Over 1 minus x y EndFraction StartFraction d x d y Over StartRoot x y EndRoot EndFraction

Expanding math expression: 1 divided by left parenthesis 1 minus x y right parenthesis as a geometric series and integrating term-by-term,

we find that

math expression: upper I equals sigma summation Underscript n equals 0 Overscript normal infinity Endscripts left parenthesis n plus one half right parenthesis Superscript negative 2 Baseline equals left parenthesis 4 minus 1 right parenthesis zeta left parenthesis 2 right parenthesis comma

providing another period representation of math expression: zeta left parenthesis 2 right parenthesis.

Changing variables:

math expression: x equals xi squared StartFraction 1 plus eta squared Over 1 plus xi squared EndFraction comma y equals eta squared StartFraction 1 plus xi squared Over 1 plus eta squared EndFraction

with Jacobian math expression: StartAbsoluteValue StartFraction partial differential left parenthesis x comma y right parenthesis Over partial differential left parenthesis xi comma eta right parenthesis EndFraction EndAbsoluteValue equals StartFraction 4 xi eta left parenthesis 1 minus xi squared eta squared right parenthesis Over left parenthesis 1 plus xi squared right parenthesis left parenthesis 1 plus eta squared right parenthesis EndFraction equals 4 StartFraction left parenthesis 1 minus x y right parenthesis StartRoot x y EndRoot Over left parenthesis 1 plus xi squared right parenthesis left parenthesis 1 plus eta squared right parenthesis EndFraction, we find

math expression: upper I equals 4 double integral Underscript 0 less than eta comma xi less than or equals 1 Endscripts StartFraction d xi Over 1 plus xi squared EndFraction StartFraction d eta Over 1 plus eta squared EndFraction equals 2 integral Subscript 0 Superscript normal infinity Baseline StartFraction d xi Over 1 plus xi squared EndFraction integral Subscript 0 Superscript normal infinity Baseline StartFraction d eta Over 1 plus eta squared EndFraction comma

the last equality being obtained by considering the involution math expression: left parenthesis xi comma eta right parenthesis right arrow from bar left parenthesis xi Superscript negative 1 Baseline comma eta Superscript negative 1 Baseline right parenthesis and comparing this with the last integral representation of math expression: pi above we obtain:

math expression: upper I equals StartFraction pi squared Over 2 EndFraction

So, we have shown that math expression: StartFraction pi squared Over 6 EndFraction and math expression: zeta left parenthesis 2 right parenthesis are accessible identities and equality follows.

Share
22

As taken from my upcoming textbook:

There is yet another solution to the Basel problem as proposed by Ritelli (2013). His approach is similar to the one by Apostol (1983), where he arrives at

math expression: StartLayout 1st Row  with Label left parenthesis 1 right parenthesis EndLabel sigma summation Underscript n greater than or equals 1 Endscripts StartFraction 1 Over n squared EndFraction equals StartFraction pi squared Over 6 EndFraction EndLayout

by evaluating the double integral

math expression: StartLayout 1st Row  with Label left parenthesis 2 right parenthesis EndLabel integral Subscript 0 Superscript 1 Baseline integral Subscript 0 Superscript 1 Baseline StartFraction normal d x normal d y Over 1 minus x y EndFraction period EndLayout

Ritelli evaluates in this case the definite integral shown in math expression: left parenthesis 4 right parenthesis. The starting point comes from realizing that math expression: left parenthesis 1 right parenthesis is equivalent to

math expression: StartLayout 1st Row  with Label left parenthesis 3 right parenthesis EndLabel sigma summation Underscript n greater than or equals 0 Endscripts StartFraction 1 Over left parenthesis 2 n plus 1 right parenthesis squared EndFraction equals StartFraction pi squared Over 8 EndFraction EndLayout

To evaluate the above sum we consider the definite integral

math expression: StartLayout 1st Row  with Label left parenthesis 4 right parenthesis EndLabel integral Subscript 0 Superscript normal infinity Baseline integral Subscript 0 Superscript normal infinity Baseline StartFraction normal d x normal d y Over left parenthesis 1 plus y right parenthesis left parenthesis 1 plus x squared y right parenthesis EndFraction period EndLayout

We evaluate math expression: left parenthesis 4 right parenthesis first with respect to x and then to y

math expression: StartLayout 1st Row  with Label left parenthesis 4 right parenthesis EndLabel 1st Column integral Subscript 0 Superscript normal infinity Baseline left parenthesis StartFraction 1 Over 1 plus y EndFraction integral Subscript 0 Superscript normal infinity Baseline StartFraction normal d x Over 1 plus x squared y EndFraction right parenthesis normal d y 2nd Column equals integral Subscript 0 Superscript normal infinity Baseline left parenthesis StartFraction 1 Over 1 plus y EndFraction left bracket StartFraction tangent Superscript negative 1 Baseline left parenthesis StartRoot y EndRoot x right parenthesis Over StartRoot y EndRoot EndFraction right bracket Subscript x equals 0 Superscript x equals normal infinity Baseline right parenthesis normal d y 2nd Row  with Label left parenthesis 5 right parenthesis EndLabel 1st Column Blank 2nd Column equals StartFraction pi Over 2 EndFraction integral Subscript 0 Superscript normal infinity Baseline StartFraction normal d y Over StartRoot y EndRoot left parenthesis 1 plus y right parenthesis EndFraction 3rd Row  with Label left parenthesis 5 right parenthesis EndLabel 1st Column Blank 2nd Column equals StartFraction pi Over 2 EndFraction integral Subscript 0 Superscript normal infinity Baseline StartFraction 2 u Over u left parenthesis 1 plus u squared right parenthesis EndFraction normal d u equals StartFraction pi squared Over 2 EndFraction comma EndLayout

where we used the substitution math expression: y right squiggle arrow u squared in the last step. If we reverse the order of integration one gets

math expression: StartLayout 1st Row  with Label left parenthesis 6 right parenthesis EndLabel 1st Column integral Subscript 0 Superscript normal infinity Baseline left parenthesis integral Subscript 0 Superscript normal infinity Baseline StartFraction normal d y Over left parenthesis 1 plus y right parenthesis left parenthesis 1 plus x squared y right parenthesis EndFraction right parenthesis normal d x 2nd Column equals integral Subscript 0 Superscript normal infinity Baseline StartFraction 1 Over 1 minus x squared EndFraction left parenthesis integral Subscript 0 Superscript normal infinity Baseline left parenthesis StartFraction 1 Over 1 plus y EndFraction minus StartFraction x squared Over 1 plus x squared y EndFraction right parenthesis normal d y right parenthesis normal d x 2nd Row  with Label left parenthesis 6 right parenthesis EndLabel 1st Column Blank 2nd Column equals integral Subscript 0 Superscript normal infinity Baseline StartFraction 1 Over 1 minus x squared EndFraction ln StartFraction 1 Over x squared EndFraction normal d x equals 2 integral Subscript 0 Superscript normal infinity Baseline StartFraction ln x Over x squared minus 1 EndFraction normal d x period EndLayout

Hence since math expression: left parenthesis 5 right parenthesis and math expression: left parenthesis 6 right parenthesis are the same, we have

math expression: StartLayout 1st Row  with Label left parenthesis 7 right parenthesis EndLabel integral Subscript 0 Superscript normal infinity Baseline StartFraction ln x Over x squared minus 1 EndFraction normal d x equals StartFraction pi squared Over 4 EndFraction period EndLayout

Furthermore

math expression: StartLayout 1st Row  with Label left parenthesis 7 right parenthesis EndLabel 1st Column integral Subscript 0 Superscript normal infinity Baseline StartFraction ln x Over x squared minus 1 EndFraction normal d x 2nd Column equals integral Subscript 0 Superscript 1 Baseline StartFraction ln x Over x squared minus 1 EndFraction normal d x plus integral Subscript 1 Superscript normal infinity Baseline StartFraction ln x Over x squared minus 1 EndFraction normal d x 2nd Row  with Label left parenthesis 8 right parenthesis EndLabel 1st Column Blank 2nd Column equals integral Subscript 0 Superscript 1 Baseline StartFraction ln x Over x squared minus 1 EndFraction normal d x plus integral Subscript 0 Superscript 1 Baseline StartFraction ln u Over u squared minus 1 EndFraction normal d u comma EndLayout

where we used the substitution math expression: x right squiggle arrow 1 divided by u. Combining math expression: left parenthesis 7 right parenthesis and math expression: left parenthesis 8 right parenthesis yields

math expression: StartLayout 1st Row  with Label left parenthesis 9 right parenthesis EndLabel integral Subscript 0 Superscript 1 Baseline StartFraction ln x Over x squared minus 1 EndFraction normal d x equals StartFraction pi squared Over 8 EndFraction period EndLayout

By expanding the denominator of the integrand in math expression: left parenthesis 10 right parenthesis into a geometric series and using the Monotone Convergence Theorem,

math expression: StartLayout 1st Row  with Label left parenthesis 10 right parenthesis EndLabel integral Subscript 0 Superscript 1 Baseline StartFraction ln x Over x squared minus 1 EndFraction normal d x equals integral Subscript 0 Superscript 1 Baseline StartFraction minus ln x Over 1 minus x squared EndFraction normal d x equals sigma summation Underscript n greater than or equals 0 Endscripts integral Subscript 0 Superscript 1 Baseline left parenthesis minus x Superscript 2 n Baseline ln x right parenthesis normal d x period EndLayout

Using integration by parts one can see that

math expression: StartLayout 1st Row  with Label left parenthesis 11 right parenthesis EndLabel integral Subscript 0 Superscript 1 Baseline left parenthesis minus x Superscript 2 n Baseline ln x right parenthesis normal d x equals left bracket minus StartFraction x Superscript 2 n plus 1 Baseline Over 2 n plus 1 EndFraction ln x right bracket Subscript 0 Superscript 1 Baseline plus integral Subscript 0 Superscript 1 Baseline StartFraction x Superscript 2 n Baseline Over 2 n plus 1 EndFraction normal d x equals StartFraction 1 Over left parenthesis 2 n plus 1 right parenthesis squared EndFraction EndLayout

Hence from math expression: left parenthesis 10 right parenthesis, and math expression: left parenthesis 11 right parenthesis

math expression: StartLayout 1st Row  with Label left parenthesis 12 right parenthesis EndLabel integral Subscript 0 Superscript 1 Baseline StartFraction ln x Over x squared minus 1 EndFraction normal d x equals sigma summation Underscript n greater than or equals 0 Endscripts StartFraction 1 Over left parenthesis 2 n plus 1 right parenthesis squared EndFraction comma EndLayout

which finishes the proof. math expression: white medium square

References:

Daniele Ritelli (2013), Another Proof of math expression: zeta left parenthesis 2 right parenthesis equals StartFraction pi squared Over 6 EndFraction Using Double Integrals, The American Mathematical Monthly, Vol. 120, No. 7, pp. 642-645

T. Apostol (1983), A proof that Euler missed: Evaluating math expression: zeta left parenthesis 2 right parenthesis the easy way, Math. Intelligencer 5, pp. 59–60, available at http://dx.doi.org/10.1007/BF03026576.

Share
1
  • This seems to be a reordering of Mike Spivey (math.stackexchange.com/users/2370/mike-spivey), Different methods to compute math expression: sigma summation Underscript k equals 1 Overscript normal infinity Endscripts StartFraction 1 Over k squared EndFraction, URL (version: 2011-08-13): math.stackexchange.com/q/57301
    – Job Bouwman
    Nov 26, 2016 at 14:10
21

Here is Euler's Other Proof by Gerald Kimble

math expression: \begin{align*} \frac{\pi^2}{6}&=\frac{4}{3}\frac{(\arcsin 1)^2}{2}\\ &=\frac{4}{3}\int_0^1\frac{\arc ...

Share
21

I saw this proof in an extract of the College Mathematics Journal.

Consider the Integeral : math expression: upper I equals integral Subscript 0 Superscript pi divided by 2 Baseline ln left parenthesis 2 cosine x right parenthesis d x

From math expression: 2 cosine left parenthesis x right parenthesis equals e Superscript i x Baseline plus e Superscript minus i x , we have:

math expression: integral Subscript 0 Superscript pi divided by 2 Baseline ln left parenthesis e Superscript i x Baseline plus e Superscript minus i x Baseline right parenthesis d x equals integral Subscript 0 Superscript pi divided by 2 Baseline ln left parenthesis e Superscript i x Baseline left parenthesis 1 plus e Superscript minus 2 i x Baseline right parenthesis right parenthesis d x equals integral Subscript 0 Superscript pi divided by 2 Baseline i x d x plus integral Subscript 0 Superscript pi divided by 2 Baseline ln left parenthesis 1 plus e Superscript minus 2 i x Baseline right parenthesis d x

The Taylor series expansion of math expression: ln left parenthesis 1 plus x right parenthesis equals x minus StartFraction x squared Over 2 EndFraction plus StartFraction x cubed Over 3 EndFraction minus midline horizontal ellipsis

Thus , math expression: ln left parenthesis 1 plus e Superscript minus 2 i x Baseline right parenthesis equals e Superscript minus 2 i x Baseline minus StartFraction e Superscript minus 4 i x Baseline Over 2 EndFraction plus StartFraction e Superscript minus 6 i x Baseline Over 3 EndFraction minus midline horizontal ellipsis, then for I :

math expression: upper I equals StartFraction i pi squared Over 8 EndFraction plus left bracket minus StartFraction e Superscript minus 2 i x Baseline Over 2 i EndFraction plus StartFraction e Superscript minus 4 i x Baseline Over 2 dot 4 i EndFraction minus StartFraction e Superscript minus 6 i x Baseline Over 3 dot 6 i EndFraction minus midline horizontal ellipsis right bracket Subscript 0 Superscript StartFraction pi Over 2 EndFraction

math expression: upper I equals StartFraction i pi squared Over 8 EndFraction minus StartFraction 1 Over 2 i EndFraction left bracket StartFraction e Superscript minus 2 i x Baseline Over 1 squared EndFraction minus StartFraction e Superscript minus 4 i x Baseline Over 2 squared EndFraction plus StartFraction e Superscript minus 6 i x Baseline Over 3 squared EndFraction minus midline horizontal ellipsis right bracket Subscript 0 Superscript StartFraction pi Over 2 EndFraction

By evaluating we get something like this..

math expression: upper I equals StartFraction i pi squared Over 8 EndFraction minus StartFraction 1 Over 2 i EndFraction left bracket StartFraction negative 2 Over 1 squared EndFraction minus StartFraction 0 Over 2 squared EndFraction plus StartFraction negative 2 Over 3 squared EndFraction minus midline horizontal ellipsis right bracket Subscript 0 Superscript StartFraction pi Over 2 EndFraction

Hence

math expression: integral Subscript 0 Superscript pi divided by 2 Baseline ln left parenthesis 2 cosine x right parenthesis d x equals StartFraction i pi squared Over 8 EndFraction minus i sigma summation Underscript k equals 0 Overscript normal infinity Endscripts StartFraction 1 Over left parenthesis 2 k plus 1 right parenthesis squared EndFraction

So now we have a real integral equal to an imaginary number, thus the value of the integral should be zero.

Thus, math expression: sigma summation Underscript k equals 0 Overscript normal infinity Endscripts StartFraction 1 Over left parenthesis 2 k plus 1 right parenthesis squared EndFraction equals StartFraction pi squared Over 8 EndFraction

But let math expression: sigma summation Underscript k equals 0 Overscript normal infinity Endscripts StartFraction 1 Over k squared EndFraction equals upper E .We get math expression: sigma summation Underscript k equals 0 Overscript normal infinity Endscripts StartFraction 1 Over left parenthesis 2 k plus 1 right parenthesis squared EndFraction equals three fourths upper E

And as a result

math expression: sigma summation Underscript k equals 0 Overscript normal infinity Endscripts StartFraction 1 Over k squared EndFraction equals StartFraction pi squared Over 6 EndFraction

Share
20

This popped up in some reading I'm doing for my research, so I thought I'd contribute! It's a more general twist on the usual pointwise-convergent Fourier series argument.


Consider the eigenvalue problem for the negative Laplacian math expression: script upper L on math expression: left bracket 0 comma 1 right bracket with Dirichlet boundary conditions; that is, math expression: script upper L f colon equals minus f Subscript n Superscript double prime Baseline equals lamda Subscript n Baseline f Subscript n with math expression: f Subscript n Baseline left parenthesis 0 right parenthesis equals f Subscript n Baseline left parenthesis 1 right parenthesis equals 0. Through inspection we can find that the admissible eigenvalues are math expression: lamda Subscript n Baseline equals n squared pi squared for math expression: n equals 1 comma 2 comma ellipsis

One can verify that the integral operator

math expression: script upper G f left parenthesis x right parenthesis equals integral Subscript 0 Superscript 1 Baseline upper G left parenthesis x comma y right parenthesis f left parenthesis y right parenthesis d y comma

where

math expression: upper G left parenthesis x comma y right parenthesis equals min left parenthesis x comma y right parenthesis minus x y equals one half left parenthesis minus StartAbsoluteValue x minus y EndAbsoluteValue plus x left parenthesis 1 minus y right parenthesis plus y left parenthesis 1 minus x right parenthesis right parenthesis comma

inverts the negative Laplacian, in the sense that math expression: script upper L script upper G f equals script upper G script upper L f equals f on the admissible class of functions (twice weakly differentiable, satisfying the boundary conditions). That is, G is the Green's function for the Dirichlet Laplacian. Because math expression: script upper G is a self-adjoint, compact operator, we can form an orthonormal basis for math expression: upper L squared left parenthesis left bracket 0 comma 1 right bracket right parenthesis from its eigenfunctions, and so may express its trace in two ways:

math expression: sigma summation Underscript n Endscripts l t f Subscript n Baseline comma script upper G f Subscript n Baseline g t equals sigma summation Underscript n Endscripts StartFraction 1 Over lamda Subscript n Baseline EndFraction

and

math expression: sigma summation Underscript n Endscripts l t f Subscript n Baseline comma script upper G f Subscript n Baseline g t equals integral Subscript 0 Superscript 1 Baseline sigma summation Underscript n Endscripts f Subscript n Baseline left parenthesis x right parenthesis l t upper G left parenthesis x comma dot right parenthesis comma f Subscript n Baseline g t d x equals integral Subscript 0 Superscript 1 Baseline upper G left parenthesis x comma x right parenthesis d x period

The latter quantity is

math expression: integral Subscript 0 Superscript 1 Baseline x left parenthesis 1 minus x right parenthesis d x equals one half minus one third equals one sixth period

Hence, we have that

math expression: sigma summation Underscript n Endscripts StartFraction 1 Over n squared pi squared EndFraction equals one sixth comma or sigma summation Underscript n Endscripts StartFraction 1 Over n squared EndFraction equals StartFraction pi squared Over 6 EndFraction period

Share
0
17

Consider the function math expression: pi cotangent left parenthesis pi z right parenthesis which has poles at math expression: z equals plus or minus n where n is an integer. Using the L'hopital rule you can see that the residue at these poles is 1.

Now consider the integral math expression: integral Underscript gamma Subscript upper N Endscripts StartFraction pi cotangent left parenthesis pi z right parenthesis Over z squared EndFraction d z where the contour math expression: gamma Subscript upper N is the rectangle with corners given by ±(N + 1/2) ± i(N + 1/2) so that the contour avoids the poles of math expression: cotangent left parenthesis pi z right parenthesis. The integral is bouond in the following way:

math expression: integral Underscript gamma Subscript upper N Baseline Endscripts StartAbsoluteValue StartFraction pi cotangent left parenthesis pi z right parenthesis Over z squared EndFraction EndAbsoluteValue d z less than or equals upper M a x StartAbsoluteValue left parenthesis StartFraction pi cotangent left parenthesis pi z right parenthesis Over z squared EndFraction right parenthesis EndAbsoluteValue upper L e n g t h left parenthesis gamma Subscript upper N Baseline right parenthesis

It can easily be shown that on the contour math expression: gamma Subscript upper N that math expression: pi cotangent left parenthesis pi z right parenthesis less than upper M where M is some constant. Then we have

math expression: integral Underscript gamma Subscript upper N Baseline Endscripts StartAbsoluteValue StartFraction pi cotangent left parenthesis pi z right parenthesis Over z squared EndFraction EndAbsoluteValue d z less than or equals upper M upper M a x StartAbsoluteValue StartFraction 1 Over z squared EndFraction EndAbsoluteValue upper L e n g t h left parenthesis gamma Subscript upper N Baseline right parenthesis equals left parenthesis 8 upper N plus 4 right parenthesis StartFraction upper M Over StartRoot 2 left parenthesis 1 divided by 2 plus upper N right parenthesis squared EndRoot squared EndFraction

where (8N+4) is the lenght of the contour and math expression: StartRoot 2 left parenthesis 1 divided by 2 plus upper N right parenthesis squared EndRoot is half the diagonal of math expression: gamma Subscript upper N. In the limit that N goes to infinity the integral is bound by 0 so we have

math expression: integral Underscript gamma Subscript upper N Baseline Endscripts StartFraction pi cotangent left parenthesis pi z right parenthesis Over z squared EndFraction d z equals 0

by the cauchy residue theorem we have 2πiRes(z = 0) + 2πi math expression: sigma summationResidues(z math expression: not equals 0) = 0. At z=0 we have Res(z=0)= math expression: minus StartFraction pi squared Over 3 EndFraction, and math expression: upper R e s left parenthesis z equals n right parenthesis equals StartFraction 1 Over n squared EndFraction so we have

math expression: 2 pi i upper R e s left parenthesis z equals 0 right parenthesis plus 2 pi i sigma summation upper R e s i d u e s left parenthesis z not equals 0 right parenthesis equals minus StartFraction pi squared Over 3 EndFraction plus 2 sigma summation Underscript 1 Overscript normal infinity Endscripts StartFraction 1 Over n squared EndFraction equals 0

Where the 2 in front of the residue at n is because they occur twice at +/- n.

We now have the desired result math expression: sigma summation Underscript 1 Overscript normal infinity Endscripts StartFraction 1 Over n squared EndFraction equals StartFraction pi squared Over 6 EndFraction.

Share
1
  • can you please explain why you divide math expression: upper M dot Length left parenthesis gamma Subscript upper N Baseline right parenthesis by half the diagonal of math expression: gamma Subscript upper N? The only thing I can think of is that it's some kind of bounds on math expression: upper M a x StartAbsoluteValue StartFraction 1 Over z squared EndFraction EndAbsoluteValue. But if you could explain it that would be great.
    – user100463
    May 4, 2016 at 23:15
17

I have another method as well. From skimming the previous solutions, I don't think it is a duplicate of any of them

In Complex analysis, we learn that math expression: sine left parenthesis pi z right parenthesis equals pi z normal upper Pi Subscript n equals 1 Superscript normal infinity Baseline left parenthesis 1 minus StartFraction z squared Over n squared EndFraction right parenthesis which is an entire function with simple zer0s at the integers. We can differentiate term wise by uniform convergence. So by logarithmic differentiation we obtain a series for math expression: pi cotangent left parenthesis pi z right parenthesis.

math expression: StartFraction d Over d z EndFraction ln left parenthesis sine left parenthesis pi z right parenthesis right parenthesis equals pi cotangent left parenthesis pi z right parenthesis equals StartFraction 1 Over z EndFraction minus 2 z sigma summation Underscript n equals 1 Overscript normal infinity Endscripts StartFraction 1 Over n squared minus z squared EndFraction

Therefore,

math expression: minus sigma summation Underscript n equals 1 Overscript normal infinity Endscripts StartFraction 1 Over n squared minus z squared EndFraction equals StartStartFraction pi cotangent left parenthesis pi z right parenthesis minus StartFraction 1 Over z EndFraction OverOver 2 z EndEndFraction

We can expand math expression: pi cotangent left parenthesis pi z right parenthesis as

math expression: pi cotangent left parenthesis pi z right parenthesis equals StartFraction 1 Over z EndFraction minus StartFraction pi squared Over 3 EndFraction z minus StartFraction pi Superscript 4 Baseline Over 45 EndFraction z cubed minus midline horizontal ellipsis

Thus,

math expression: \begin{align} \frac{\pi\cot(\pi z) - \frac{1}{z}}{2z} &= \frac{- \frac{\pi^2}{3}z - \frac{\pi^4}{45} ...

Share
14

I would like to present you a method I found recently here.

Let math expression: upper A Subscript n Baseline equals integral Subscript 0 Superscript pi divided by 2 Baseline cosine Superscript 2 n Baseline x normal d x and math expression: upper B Subscript n Baseline equals integral Subscript 0 Superscript pi divided by 2 Baseline x squared cosine Superscript 2 n Baseline x normal d x.

The first integral is well known, by per partes we get the recurecnce relation :

math expression: StartLayout 1st Row  with Label left parenthesis 1 right parenthesis EndLabel upper A Subscript n Baseline equals StartFraction 2 n minus 1 Over 2 n EndFraction upper A Subscript n minus 1 EndLayout

By per partes for the second integral:

math expression: upper A Subscript n Baseline equals integral Subscript 0 Superscript pi divided by 2 Baseline cosine Superscript 2 n Baseline x normal d x equals x cosine Superscript 2 n Baseline x vertical bar Subscript 0 Superscript pi divided by 2 Baseline minus StartFraction x squared Over 2 EndFraction left parenthesis cosine Superscript 2 n Baseline x right parenthesis Superscript prime Baseline vertical bar Subscript 0 Superscript pi divided by 2 Baseline plus one half integral Subscript 0 Superscript pi divided by 2 Baseline x squared left parenthesis cosine Superscript 2 n Baseline x right parenthesis Superscript double prime Baseline normal d x

First two terms vanish, so we are left only with the integral and since math expression: left parenthesis cosine Superscript 2 n Baseline x right parenthesis double prime equals 2 n left parenthesis 2 n minus 1 right parenthesis cosine Superscript 2 n minus 2 Baseline x minus 4 n squared cosine Superscript 2 n Baseline x we have :

math expression: StartLayout 1st Row  with Label left parenthesis 2 right parenthesis EndLabel upper A Subscript n Baseline equals left parenthesis 2 n minus 1 right parenthesis n upper B Subscript n minus 1 Baseline minus 2 n squared upper B Subscript n EndLayout

for math expression: n greater than or equals 1. Rearranging and substituing math expression: left parenthesis 2 n minus 1 right parenthesis equals 2 n StartFraction upper A Subscript n Baseline Over upper A Subscript n minus 1 Baseline EndFraction from math expression: left parenthesis 1 right parenthesis we get :

math expression: StartLayout 1st Row  with Label left parenthesis 3 right parenthesis EndLabel StartFraction 1 Over n squared EndFraction equals 2 StartFraction upper B Subscript n minus 1 Baseline Over upper A Subscript n minus 1 Baseline EndFraction minus 2 StartFraction upper B Subscript n Baseline Over upper A Subscript n Baseline EndFraction EndLayout

Summing from 1 to some k natural we get by telescoping property

math expression: StartLayout 1st Row  with Label left parenthesis 4 right parenthesis EndLabel sigma summation Underscript n equals 1 Overscript k Endscripts StartFraction 1 Over n squared EndFraction equals 2 StartFraction upper B 0 Over upper A 0 EndFraction minus 2 StartFraction upper B Subscript k Baseline Over upper A Subscript k Baseline EndFraction equals StartFraction pi squared Over 6 EndFraction minus 2 StartFraction upper B Subscript k Baseline Over upper A Subscript k Baseline EndFraction EndLayout

Next, using the inequality math expression: sine x greater than or equals StartFraction 2 x Over pi EndFraction on math expression: left parenthesis 0 comma StartFraction pi Over 2 EndFraction right parenthesis and by math expression: left parenthesis 1 right parenthesis :

math expression: StartFraction 4 Over pi squared EndFraction upper B Subscript n minus 1 Baseline equals StartFraction 4 Over pi squared EndFraction integral Subscript 0 Superscript pi divided by 2 Baseline x squared cosine Superscript 2 n minus 2 Baseline x normal d x less than integral Subscript 0 Superscript pi divided by 2 Baseline sine squared x cosine Superscript 2 n minus 2 Baseline x normal d x equals upper A Subscript n minus 1 Baseline minus upper A Subscript n Baseline equals StartFraction upper A Subscript n minus 1 Baseline Over 2 n EndFraction

so in the limit the last term vanishes by the sqeeze theorem, so we are left with

math expression: StartLayout 1st Row  with Label left parenthesis 4 right parenthesis EndLabel sigma summation Underscript n equals 1 Overscript normal infinity Endscripts StartFraction 1 Over n squared EndFraction equals StartFraction pi squared Over 6 EndFraction EndLayout

That concludes the result.

Share
2
  • I think you mean math expression: sine x greater than or equals 2 x divided by pi. But this is a clever approach. Another reason to appreciate integration by parts.
    – user123641
    Jun 9, 2017 at 0:51
  • Thanks for sharing, very easy to follow and totally new to me. Fixed the typo mentioned by @Bryan.
    – AD - Stop Putin -
    Jun 9, 2017 at 10:11
14

Applying the usual trick 1 transforming a series to an integral, we obtain

math expression: sigma summation Underscript n equals 1 Overscript normal infinity Endscripts StartFraction 1 Over n squared EndFraction equals integral Subscript 0 Superscript 1 Baseline integral Subscript 0 Superscript 1 Baseline StartFraction d x d y Over 1 minus x y EndFraction

where we use the Monotone Convergence Theorem to integrate term-wise.

Then there's this ingenious change of variables 2, which I learned from Don Zagier during a lecture, and which he in turn got from a colleague:

math expression: left parenthesis x comma y right parenthesis equals left parenthesis StartFraction cosine v Over cosine u EndFraction comma StartFraction sine u Over sine v EndFraction right parenthesis comma 0 less than or equals u less than or equals v less than or equals StartFraction pi Over 2 EndFraction

One verifies that it is bijective between the rectangle math expression: left bracket 0 comma 1 right bracket squared and the triangle math expression: 0 less than or equals u less than or equals v less than or equals StartFraction pi Over 2 EndFraction, and that its Jacobian determinant is precisely math expression: 1 minus x squared y squared, which means math expression: StartFraction 1 Over 1 minus x squared y squared EndFraction would be a neater integrand. For the moment, we have found

math expression: upper J equals integral Subscript 0 Superscript 1 Baseline integral Subscript 0 Superscript 1 Baseline StartFraction d x d y Over 1 minus x squared y squared EndFraction equals StartFraction pi squared Over 8 EndFraction

(the area of the triangular domain in the math expression: left parenthesis u comma v right parenthesis plane).


There are two ways to transform math expression: integral StartFraction 1 Over 1 minus x y EndFraction into something math expression: integral StartFraction 1 Over 1 minus x squared y squared EndFractionish:

  • Manipulate math expression: upper S equals sigma summation Underscript n equals 1 Overscript normal infinity Endscripts StartFraction 1 Over n squared EndFraction: We have math expression: sigma summation Underscript n equals 1 Overscript normal infinity Endscripts StartFraction 1 Over left parenthesis 2 n right parenthesis squared EndFraction equals one fourth upper S so math expression: sigma summation Underscript n equals 0 Overscript normal infinity Endscripts StartFraction 1 Over left parenthesis 2 n plus 1 right parenthesis squared EndFraction equals three fourths upper S. Applying the series-integral transformation, we get math expression: three fourths upper S equals upper J so

    math expression: upper S equals StartFraction pi squared Over 6 EndFraction

  • Manipulate math expression: upper I equals integral Subscript 0 Superscript 1 Baseline integral Subscript 0 Superscript 1 Baseline StartFraction d x d y Over 1 minus x y EndFraction: Substituting math expression: left parenthesis x comma y right parenthesis left arrow left parenthesis x squared comma y squared right parenthesis we have math expression: upper I equals integral Subscript 0 Superscript 1 Baseline integral Subscript 0 Superscript 1 Baseline StartFraction 4 x y d x d y Over 1 minus x squared y squared EndFraction so

    math expression: upper J equals integral Subscript 0 Superscript 1 Baseline integral Subscript 0 Superscript 1 Baseline StartFraction d x d y Over 1 minus x squared y squared EndFraction equals integral Subscript 0 Superscript 1 Baseline integral Subscript 0 Superscript 1 Baseline StartFraction left parenthesis 1 plus x y minus x y right parenthesis d x d y Over 1 minus x squared y squared EndFraction equals upper I minus one fourth upper I

    whence

    math expression: upper I equals four thirds upper J equals StartFraction pi squared Over 6 EndFraction

(It may be seen that they are essentially the same methods.)


After looking at the comments it seems that this looks a lot like Proof 2 in the article by R. Chapman.

See also: Multiple Integral math expression: integral Subscript 0 Superscript 1 Baseline integral Subscript 0 Superscript 1 Baseline integral Subscript 0 Superscript 1 Baseline integral Subscript 0 Superscript 1 Baseline StartFraction 1 Over 1 minus x y u v EndFraction d x d y d u d v

1 See e.g. Proof 1 in Chapman's article.
2 It may have been a different one; maybe as in the above article. Either way, the idea to do something trigonometric was not mine.

Share
1
  • 2
    The second proof is by Beukers, Kolk and Calabi, which is in here pdfs.semanticscholar.org/35be/…
    – Vivek Kaushik
    Jun 28, 2017 at 15:19
14

This is, by no measure, the best nor the simplest approach, but I think the approach is pretty peculiar.

We estimate the number math expression: upper N left parenthesis x right parenthesis of integer solutions to math expression: a squared plus b squared plus c squared plus d squared less than or equals x as math expression: x right arrow normal infinity. On one hand, this is the number of lattice points inside the the 4-ball of radius math expression: StartRoot x EndRoot, which has volume math expression: one half pi squared x squared, hence math expression: upper N left parenthesis x right parenthesis equals StartFraction pi squared Over 2 EndFraction x squared plus upper O left parenthesis x Superscript 3 divided by 2 Baseline right parenthesis.

On the other hand, let math expression: r 4 left parenthesis n right parenthesis be the number of solutions to math expression: a squared plus b squared plus c squared plus d squared equals n. Following the derivation in the book by Iwaniec-Kowalski, by Jacobi's four-square identity we can write

math expression: upper N left parenthesis x right parenthesis equals sigma summation Underscript n less than or equals x Endscripts r 4 left parenthesis n right parenthesis equals 8 sigma summation Underscript m less than or equals x Endscripts left parenthesis 2 plus left parenthesis negative 1 right parenthesis Superscript m Baseline right parenthesis sigma summation Underscript d m less than or equals x comma d odd Endscripts d equals 8 sigma summation Underscript m less than or equals x Endscripts left parenthesis 2 plus left parenthesis negative 1 right parenthesis Superscript m Baseline right parenthesis left parenthesis StartFraction x squared Over 4 m squared EndFraction plus upper O left parenthesis StartFraction x Over m EndFraction right parenthesis right parenthesis equals 2 x squared sigma summation Underscript m less than or equals x Endscripts left parenthesis 2 plus left parenthesis negative 1 right parenthesis Superscript m Baseline right parenthesis m Superscript negative 2 Baseline plus upper O left parenthesis x log x right parenthesis equals 3 x squared zeta left parenthesis 2 right parenthesis plus upper O left parenthesis x log x right parenthesis

(I have copied the steps as they were in the book, it's a neat exercise to justify every transition). In particular, we have

math expression: zeta left parenthesis 2 right parenthesis equals limit Underscript x right arrow normal infinity Endscripts StartFraction upper N left parenthesis x right parenthesis Over 3 x squared EndFraction equals StartFraction pi squared Over 6 EndFraction period

Share
1
  • (+1) I wonder if one can do the same by only exploiting the fact that the average value of math expression: r 2 left parenthesis n right parenthesis is math expression: pi by Gauss circle problem.
    – Jack D'Aurizio
    Nov 9, 2017 at 4:50
13

See evaluations of Riemann Zeta Function math expression: zeta left parenthesis 2 right parenthesis equals sigma summation Underscript n equals 1 Overscript normal infinity Endscripts StartFraction 1 Over n squared EndFraction in mathworld.wolfram.com and a solution by in D. P. Giesy in Mathematics Magazine:

D. P. Giesy, Still another elementary proof that math expression: sigma summation Underscript n equals 1 Overscript normal infinity Endscripts StartFraction 1 Over n squared EndFraction equals StartFraction pi squared Over 6 EndFraction, Math. Mag. 45 (1972) 148–149.

Unfortunately I did not get a link to this article. But there is a link to a note from Robin Chapman seems to me a variation of proof's Giesy.

Share
1
2 Next